168

Click here to load reader

эл курс подг к егэ Word (2)

Embed Size (px)

DESCRIPTION

 

Citation preview

Page 1: эл курс подг к егэ Word (2)

Программа элективного курса по подготовке к Единому государственному экзамену по химии

( 30 часов) Занятие 1. (1 час) Кодификатор элементов содержания по химии для составления контрольных измерительных материалов единого государственного экзамена 2007 годСтруктура экзаменационной работы.Характеристика заданий «А» и особенности их выполнения. Характеристика заданий «В» и особенности их выполнения. Характеристика заданий «С» и особенности их выполнения. Обобщенный план экзаменационной работы по химии. Спецификация экзаменационной работы по химии.Анализ наиболее трудных вопросов ЕГЭ по химии.

Занятие 2. Входной контроль (3 часа)

Занятие 3. Строение атома (1 час) Атом. Состав атомных ядер. Современные представления о строении атомов. Химический элемент. Формы существования химических элементов. Основное и возбужденное состояние атомов. Изотопы. Относительная атомная и относительная молекулярная масса. Строение электронных оболочек атомов элементов первых четырех периодов. Понятие об электронном облаке, s- и p-электронах. Радиусы атомов, их периодические изменения в системе химических элементов.Контрольное тестирование.

Занятие 4. Периодический закон и периодическая система химических элементов Д.И.Менделеева (1 час)

Периодический закон химических элементов Д.И. Менделеева. Малые и большие периоды, группы и подгруппы. Закономерности изменения химических свойств элементов и их соединений по периодам и группам.Контрольное тестирование.

Занятие 5. Виды химической связи. Кристаллические решетки. (1час)Виды химической связи: ковалентная (полярная и неполярная). Способы образования ковалентной связи. Характеристики ковалентной связи: длина и энергия связи Ионная связь и ее образование. Металлическая и водородная связь. Примеры соединений со связями разных типов. Вещества молекулярного и немолекулярного строения. Зависимость свойств веществ от особенностей их кристаллических решеток.

3

Page 2: эл курс подг к егэ Word (2)

Контрольное тестирование.Занятие 6. Валентность и степень окисления. (1 час)Электроотрицательность химических элементов. Заряды ионов. Степень окисления.Контрольное тестирование.

Занятие 7. Химические реакции (1 час)Типы химических реакций: реакции соединения, разложения, замещения, ионного обмена. Окислительно-восстановительные реакции в неорганической и органической химии. Окислитель, восстановитель. Контрольное тестирование.

Занятие 8. Тепловой эффект химической реакции (1 час)

Тепловой эффект химических реакций. Сохранение и превращение энергии при химических реакциях. Расчёты теплового эффекта химической реакции.Контрольное тестирование.

Занятие 9. Скорость химической реакции. Химическое равновесие(1 час)

Скорость химических реакций. Зависимость скорости от природы реагирующих веществ, концентрации, температур. Катализ.Обратимые и необратимые химические реакции. Химическое равновесие и условия его смещения.Контрольное тестирование.

Занятие 10. Растворы. Электролитическая диссоциация (2 часа) Растворы. Концентрация растворов. Электролитическая диссоциация неорганических и органических кислот, щелочей, солей. Степень диссоциации. Сильные и слабые электролиты.Контрольное тестирование.

Занятие 11. Важнейшие классы неорганических соединений (1 час) Классификация неорганических веществ. Характерные химические свойства металлов и неметаллов.Оксиды основные, кислотные и амфотерные. Номенклатура, способы получения, физические и химические свойства оксидов.Основания, их классификация, номенклатура, способы получения, физические и химические свойства. Амфотерные гидроксиды, способы получения и химические свойства.Кислоты, их классификация, номенклатура, способы получения, физические и химические свойства.

4

Page 3: эл курс подг к егэ Word (2)

Соли средние и кислые. Химические свойства солей. Способы получения солей. Гидролиз солей. Электролиз расплавов и растворов солей.Реакции, подтверждающие взаимосвязь различных классов неорганических веществ. Контрольное тестирование.

Занятие 12. Гидролиз неорганических и органических соединений (1 час)

Гидролиз солей. Усиление и ослабление гидролиза солей.Гидролиз органических соединений. Гидролиз простых и сложных неорганических соединенийКонтрольное тестирование.

Занятие 13. Металлы (2 часа)Металлы. Положение металлов в периодической системе. Особенности строения атомов металлов. Общая характеристика металлов главных подгрупп I—III групп в связи с их положением в периодической системе химических элементов Д.И. Менделеева и особенностями строения их атомов.Характеристика переходных элементов – меди, цинка, хрома, железа по их положению в периодической системе химических элементов Д.И. Менделеева и особенностям строения их атомов. Расчёты массовых долей компонентов смеси.Контрольное тестирование.

Занятие 14. Неметаллы (2 часа) Общая характеристика неметаллов главных подгрупп IV- VII групп по положению в периодической системе химических элементов Д.И. Менделеева и особенности строения их атомов.Углерод, его аллотропные формы. Соединения углерода: оксиды углерода (II и IV), угольная кислота и её соли. Кремний и его соединения. Химические свойства, нахождение в природе.Кислород. Аллотропные формы. Физические и химические свойства. Получение кислорода.Сера, её физические свойства и химические свойства. Соединения серы: сероводород, оксиды серы. Серная кислота, её свойства.Азот. Физические и химические свойства. Соединения азота: Аммиак, соли аммония. Оксиды азота, азотная кислота и её соли (физические и химические свойства).Фосфор, его аллотропные формы, физические и химические свойства. Оксиды фосфора (V), фосфорная кислота и её соли.Водород. Физические и химические свойства. Получение водорода.

5

Page 4: эл курс подг к егэ Word (2)

Взаимодействие водорода с кислородом, оксидами металлов, с органическими веществами. Контрольное тестирование. Занятие 15. Углеводороды (2 часа)Предельные углеводороды. Гомологический ряд предельных углеводородов, их электронное и пространственнее строение(sp3-гибридизация). Метан. Номенклатура, физические и химические свойства предельных углеводородов. Механизм реакции замещения. Циклопарафины.Непредельные углеводороды. Гомологический ряд этиленовых углеводородов. Двойная связь. (σ- и π- связи), sp2 – гибридизация, Физические свойства. Структурная и пространственная изомерия. Номенклатура этиленовых углеводородов. Химические свойства. Механизм реакции присоединения. Правило В.В. Марковникова. Получение этиленовых углеводородов.Ацетилен. Тройная связь, sp-гибридизация. Гомологический ряд ацетилена. Физические и химические свойства, получение.Ароматические углеводороды. Бензол, его электронное строение, химические свойства.Природные источники углеводородов. Нефть, природный газ, уголь. Фракционная перегонка нефти. Крекинг. Ароматизация нефтепродуктов.Контрольное тестирование.

Занятие 16. Кислородсодержащие органические соединения ( 2 часа)Спирты. Строение, физические свойства. Электронное строение функциональной группы. Изомерия. Номенклатура спиртов. Химические свойства спиртов. Получение спиртов.Фенол, его строение, физические и химические свойства фенола.Альдегиды. Электронное строение альдегидной группы. Строение и химические свойства альдегидов. Получение альдегидов.Карбоновые кислоты.Гомологический ряд предельных одноосновных карбоновых кислот, их строение. Карбоксильная группа, взаимное влияние карбоксильной группы и углеводородного радикала. Физические и химические свойства карбоновых кислот. Уксусная, муравьиная, пальмитиновая, стеариновая, олеиновая кислоты. Получение кислот.Сложные эфиры. Жиры.Сложные эфиры. Строение, получение реакцией этерификации. Химические свойства. Углеводы.Глюкоза, её строение, химические свойства.

6

Page 5: эл курс подг к егэ Word (2)

Сахароза, гидролиз сахарозы. Крахмал и целлюлоза, их строение, химические свойства, Применение целлюлозы и её производных. Понятие об искусственных волокнах. Реакции, подтверждающие взаимосвязь углеводородов и кислородсодержащих органических веществ.Контрольное тестирование.Занятие 17. Азотсодержащие органические соединения (2 часа)Амины как органические основания Строение, аминогруппа. Химические свойства аминов. Получение аминов жирного ряда.Анилин. Получение, Химические свойства.Аминокислоты. Строение, химические особенности , изомерия аминокислот. Белки. Строение, структура и свойства белков. Общие понятия химии высокомолекулярных соединений: мономер, полимер, структурное звено, степень полимеризации, средняя молекулярная масса. Полимеризация, поликонденсация.Контрольное тестирование. Занятие 18-19. Выполнение заданий части С (2часа)Занятие 20. Выходной контроль (3 часа)

Рекомендуемая литератураЛидин Р.А. и др. Руководство для подготовки к экзаменам.- М.: АСТ: Астрель: Профиздат, 2005.

Лидин Р.А. и др. Химия 10-11. Учебное пособие. – М.: Дрофа, 2006.

Кузьменко Н.Е., Еремин В. В. Химия : для школьников старших классов и поступающих в вузы: Учебное пособие: -М.: Дрофа, 2002.

Химия: задачи с ответами и решениями: Учеб.- метод.-пособие/ Под ред. Проф. Т. В. Лисичкина. – М.: Новая волна, 2004.

Егоров А. С., Химия: экспресс – репетитор для поступающих в вузы. – Ростов н/Д: Феникс. 2008.

Косова О. Ю., Единый государственный экзамен. Химия: Справочные материалы, контрольно – тренировочные упражнения, расчетные задачи. – Челябинск: Взгляд, 2004, 2005,2006.

7

Page 6: эл курс подг к егэ Word (2)

Занятие 1

Кодификатор элементов содержания по химии для составления контрольных измерительных материалов

единого государственного экзамена 2009 г.Кодификатор … будет служить планом подготовки к ЕГЭ по химии, все занятия будут строиться согласно кодификатору …

Жирным курсивом указаны крупные блоки содержания. Отдельные элементы содержания, на основе которых составляют проверочные задания, обозначены в блоках кодом контролируемого элемента.

Код раздела

Кодконт-роли-

руемого элемента

Элементы содержания, проверяемые заданиями КИМ

1 Химический элемент

1.1 Современные представления о строении атомов. Изотопы. Строение электронных оболочек атомов элементов первых четырех периодов: s-, p- и -d элементы. Электронная конфигурация атома. Основное и возбужденное состояние атомов.

1.2 Периодический закон и периодическая система химических элементов Д.И. Менделеева. Радиусы атомов, их периодические изменения в системе химических элементов. Закономерности изменения химических свойств элементов и их соединений по периодам и группам.

2 Вещество

2.1 Химическая связь: ковалентная (полярная и неполярная), ионная, металлическая, водородная. Способы образования ковалентной связи. Характеристики ковалентной связи: длина и энергия связи. Образование ионной связи.

2.2 Электроотрицательность. Степень окисления и валентность химических элементов.

2.3 Вещества молекулярного и немолекулярного строения. Зависимость свойств веществ от особенностей их кристаллической решетки.

2.4 Классификация неорганических веществ.2.5 Общая характеристика металлов главных подгрупп I—

III групп в связи с их положением в периодической системе химических элементов Д.И. Менделеева и особенностями строения их атомов.

8

Page 7: эл курс подг к егэ Word (2)

2.6 Характеристика переходных элементов – меди, цинка, хрома, железа по их положению в периодической системе химических элементов Д.И. Менделеева и особенностям строения их атомов.

2.7 Общая характеристика неметаллов главных подгрупп IV-VII групп в связи с их положением в периодической системе химических элементов Д.И. Менделеева и особенностями строения их атомов.

2.8 Характерные химические свойства простых веществ – металлов: щелочных, щелочноземельных, алюминия, переходных металлов – меди, цинка, хрома, железа.

2.9

Характерные химические свойства простых веществ – неметаллов: водорода, галогенов, кислорода, серы, азота, фосфора, углерода, кремния.

2.10 Характерные химические свойства оксидов: основных, амфотерных, кислотных.

2.11 Характерные химические свойства оснований и амфотерных гидроксидов.

2.12 Характерные химические свойства кислот.2.13 Характерные химические свойства солей: средних,

кислых, основных; комплексных (на примере соединений алюминия и цинка).

2.14 Теория строения органических соединений. Изомерия – структурная и пространственная. Гомология.

2.15 Типы связей в молекулах органических веществ. Гибридизация атомных орбиталей углерода. Радикал. Функциональная группа.

2.16 Классификация и номенклатура органических соединений.

2.17 Характерные химические свойства углеводородов: алканов, алкенов, диенов, алкинов.

2.18 Характерные химические свойства ароматических углеводородов: бензола и толуола.

2.19 Характерные химические свойства предельных одноатомных и многоатомных спиртов; фенола.

2.20 Характерные химические свойства альдегидов, предельных карбоновых кислот, сложных эфиров.

2.21 Характерные химические свойства азотсодержащих органических соединений: аминов и аминокислот.

2.22 Биологически важные вещества: жиры, углеводы (моносахариды, дисахариды, полисахариды), белки.

2.232.23.12.23.2

Взаимосвязь различных классов:неорганических веществ;органических веществ.

9

Page 8: эл курс подг к егэ Word (2)

3 Химическая реакция

3.1 Классификация химических реакций в неорганической и органической химии.

3.2 Скорость реакции, ее зависимость от различных факторов.

3.3 Тепловой эффект химической реакции. Термохимические уравнения.

3.4 Обратимые и необратимые химические реакции. Химическое равновесие. Смещение равновесия под действием различных факторов.

3.5 Диссоциация электролитов в водных растворах. Слабые и сильные электролиты.

3.6 Реакции ионного обмена.3.7 Реакции окислительно-восстановительные. Коррозия

металлов и способы защиты от нее.3.8 Гидролиз солей. Среда водных растворов: кислая,

нейтральная, щелочная.3.9 Электролиз расплавов и растворов (солей, щелочей).3.10 Механизмы реакций замещения и присоединения в

органической химии. Правило В.В. Марковникова3.11

3.11.13.11.2

Реакции, характеризующие основные свойства и способы получения:углеводородов;кислородосодержащих соединений;

3.12

3.12.13.12.2

Реакции, подтверждающие взаимосвязь различных классов:неорганических веществ;углеводородов и кислородосодержащих органических соединений.

4Познание и применение веществ и химических

реакций

4.1 Правила работы в лаборатории. Лабораторная посуда и оборудование. Правила безопасности при работе с едкими, горючими и токсичными веществами, средствами бытовой химии.

4.2 Методы исследования объектов, изучаемых в химии. Определение характера среды водных растворов веществ. Индикаторы.Качественные реакции на неорганические вещества и ионы, отдельные классы органических соединений.

4.3 Общие способы получения металлов. Общие научные принципы химического производства (на примере промышленного получения аммиака, серной и азотной

10

Page 9: эл курс подг к егэ Word (2)

кислот, чугуна, стали, метанола). Промышленное получение веществ и охрана окружающей среды.

4.4 Природные источники углеводородов, их переработка.4.5 Основные методы синтеза высокомолекулярных

соединений (пластмасс, синтетических каучуков, волокон).

4.6 Вычисление массы растворенного вещества, содержащегося в определенной массе раствора с известной массовой долей.

4.7 Расчеты: объемных отношений газов при химических реакциях.

4.8 Расчеты: массы вещества или объема газов по известному количеству вещества, массе или объему одного из участвующих в реакции веществ.

4.9 Расчеты: теплового эффекта реакции.4.10 Расчеты: массы (объема, количества вещества)

продуктов реакции, если одно из веществ дано в избытке (имеет примеси).

4.11 Расчеты: массы (объема, количества вещества) продукта реакции, если одно из веществ дано в виде раствора с определенной массовой долей растворенного вещества.

4.12 Нахождение молекулярной формулы вещества.4.13 Расчеты: массовой или объемной доли выхода

продукта от теоретически возможного.4.14 Расчеты: массовой доли (массы) химического

соединения в смеси

Ознакомление со структурой экзаменационной работы по химии

Каждый вариант экзаменационной работы состоит из трех частей и включает 45 заданий. Одинаковые по форме представления и уровню сложности задания сгруппированы в определенной части работы.

Часть 1 содержит 30 заданий с выбором ответа (базового уровня сложности). Их обозначение в работе: А1; А2; А3; А4 … А30.

Задания с выбором ответа, самые многочисленные в экзаменационной работе, построены на материале практически всех важнейших разделов школьного курса химии. В своей совокупности они проверяют на базовом уровне усвоение значительного количества элементов содержания, предусмотренных стандартом образования (43 из 56) из всех четырех содержательных блоков курса – «Химический элемент», «Вещество», «Химическая реакция», «Познание и применение веществ и химических реакций».

11

Page 10: эл курс подг к егэ Word (2)

Выполнение заданий с выбором ответа предполагает использование знаний для подтверждения правильности одного из четырех предложенных вариантов ответа. Последовательное соотнесение каждого из предложенных вариантов ответа с условием задания – основное правило, которое должно соблюдаться при выполнении этих заданий.

Пример 1. Оксид с наиболее ярко выраженными оснóвными свойствами образует элемент, химический знак которого:

1) Be 2) Mg 3) Ba 4) ZnРешение.Сравним радиусы атомов представленных химических элементов по их положению в периодической системе. Самый большой радиус у атома бария, значит он легче всего отдает электроны с внешнего энергетического уровня, следовательно, проявляет наиболее ярко выраженные металлические свойства и ему соответствует основный оксид.

Правильный ответ: 3Пример 2. В схеме превращений

Na2CO3 MgCO3 Mg(HCO3)2

буквами «Х» и «Y» обозначены вещества1) X – MgO; Y – HCl 2) X – Mg(OH)2; Y – NaCl 3) X – MgF2; Y – CO2 и H2O 4) X – MgCl2; Y – CO2 и H2O Решение.Карбонат натрия может взаимодействовать только с раствором хлорида магния с образованием карбоната магния.

Правильный ответ: 4Пример 3. При обычных условиях с наибольшей скоростью протекает реакция1) Cu + O2 = 2CuO 3) Fe + 2HCl = FeCl2+ H2

2) BaCl2+ Na2CO3 = BaCO3↓ + 2NaCl 4) Cu + S = CuS

Решение. С наибольшей скоростью протекают реакции ионного обмена. Следуя этому правилу, выбираем реакцию ионного обмена. Это реакция между хлоридом бария и карбонатом натрия (Ba2+ +CO3

2- = BaCO3↓). Правильный ответ: 2

Пример 4. Согласно уравнению OH- + H+ = H2O возможно химическое взаимодействие между веществами1) Cu(OH)2 + 2HCl = CuCl2 + 2H2O2) Ba(OH)2 + H2SO4 = BaSO4 + 2H2O3) Ba(OH)2 + 2HCl = BaCl2 + 2H2O

12

Page 11: эл курс подг к егэ Word (2)

4) 2KOH + H2SiO3 = K2SiO3 + 2H2O

Решение.Из краткого ионного уравнения OH- + H+ = H2O следует, что в реакцию должны вступить сильная кислота и сильное основание. Поэтому сразу исключаем первый и четвёртый ответы. Из оставшихся вариантов ответов выбираем тот, в правой части которого нет больше неэлектролитов, кроме воды, то есть ответ 3, так как во втором случае кроме воды образуется и осадок BaSO4

Правильный ответ: 3Пример 5.

Гомологом бензола является вещество, молекулярная формула которого:

1) С8Н18 2) С8Н10 3) С8Н16 4) С8Н14

Решение.Гомологический ряд бензола отвечает общей формуле СnН2n-6. подставляем значение n = 8 и получаем число атомов водорода: 2·8-6=10. Молекулярная формула гомолога С8Н10.

Правильный ответ: 2

Часть 2 содержит 10 заданий с кратким ответом (повышенного уровня сложности). Их обозначение в работе: В1; В2; В3 … В10.

Задания с кратким ответом также построены на материале важнейших разделов курса химии, но в отличие от заданий с выбором ответа имеют повышенный уровень сложности. Это проявляется прежде всего в том, что выполнение таких заданий предполагает:

а) осуществление большего числа учебных действий, чем в случае заданий с выбором ответа;

б) самостоятельное формулирование и запись ответа.

В экзаменационной работе предложены следующие разновидности заданий с кратким ответом:

1. Задания на установление соответствия позиций, представленных в двух множествах.

Пример 6. Установите соответствие между исходными веществами и продуктами окислительно-восстановительных реакций (ОВР).

13

Page 12: эл курс подг к егэ Word (2)

ИСХОДНЫЕ ВЕЩЕСТВА ПРОДУКТЫ ОВР А) Fe + Cl2 = 1) FeSO4 + H2

Б) Fe + HCl = 2) Fe2(SO4)3 + H2

В) Fe + H2SO4 (разб.) = 3) Fe2(SO4)3 + SO2 + H2O Г) Fe + H2SO4 (конц.) = 4) FeCl2 + H2

5) FeCl3 + H2

6) FeCl3

Решение.Хлор относится к сильным окислителям и окисляет железо до степени окисления +32Fe + 3Cl2 = 2FeCl3 (ответ 6)Соляная кислота более слабый окислитель, поэтому железо окисляет до степени окисления +2Fe + 2HCl= FeCl2 + H2 (ответ 4)Аналогично действует разбавленная серная кислота (степень окисления железа +2)Fe + H2SO4(разб) = FeSO4 + H2 (ответ 1)Концентрированная серная кислота является довольно сильным окислителем за счёт серы + 6, особенно при нагревании2Fe + 6H2SO4 (конц) Fe2(SO4)3 + 3SO2 + 6H2O (ответ 3)

Правильный ответ: 6413

Пример 7. Установите соответствие между названием соли и типом гидролиза её водного раствора

НАЗВАНИЕ СОЛИ ТИП ГИДРОЛИЗАА) сульфид алюминия 1) по аниону Б) нитрат железа (II) 2) по катиону В) сульфит натрия 3) по катиону и аниону Г) хлорид аммония

Правильный ответ: 32122. Задания на выбор нескольких правильных ответов из предложенного перечня ответов (множественный выбор).

Пример 8. Глицин может реагировать с: 1) сульфатом калия 2) гидроксидом калия 3) метаном 4) бромоводородом 5) этаналем 6) аланиномРешение. Глицин (аминоуксусная кислота), как все аминокислоты является амфотерным соединением, что обусловлено наличием в их молекулах функциональных групп кислотного ( – СООН) и основного (–

14

Page 13: эл курс подг к егэ Word (2)

NH2) характера. По этой причине аминокислоты реагируют и с кислотами и с основаниями. Аминокислоты реагируют друг с другом за счет взаимодействия карбоксильной группы одной молекулы и аминогруппы другой молекулы аминокислоты, образуя дипептиды.

Правильный ответ: 246

3. Задания, требующие написания ответа в виде числа.

Пример 9.

К 200г 20%-ного раствора нитрата натрия добавили 10г этой же соли. Рассчитайте массовую долю нитрата натрия в полученном растворе.Ответ: __________ %. (Запишите число с точностью до целых.)

Правильный ответ: 24

Часть 3 содержит 5 заданий с развернутым ответом (высокого уровня сложности). Их обозначение в работе: С1; С2; С3; С4; С5.

Задания с развернутым ответом ориентированы на проверку умений:

– объяснять обусловленность свойств и применения веществ их составом и строением; характер взаимного влияния атомов в молекулах органических соединений; взаимосвязь неорганических и органических веществ; сущность и закономерность протекания изученных типов реакций;

– проводить комбинированные расчеты по химическим уравнениям и по определению молекулярной формулы вещества.

Задания с развернутым ответом – самые сложные в экзаменационной работе. В отличие от заданий с выбором ответа и кратким ответом, они предусматривают одновременную проверку усвоения нескольких (двух и более) элементов содержания из различных содержательных блоков и подразделяются на следующие типы:

– задания, проверяющие усвоение основополагающих элементов содержания, таких, например, как «окислительно-восстановительные реакции»;

Пример 10. Используя метод электронного баланса, составьте уравнение реакции:

K2Cr2O7 + HCI ® CrCI3 + …+ … + H2OОпределите окислитель и восстановитель.Решение. Cоставляем электронный баланс:

3 2CI– –2ē → CI20

1 2Cr+6 +6ē → 2Cr+3

15

Page 14: эл курс подг к егэ Word (2)

2) Расставляем коэффициенты в уравнении реакции:K2Cr2O7 + 14HCI = 2CrCI3 + CI2 +2KCI + 7H2O

– задания, проверяющие усвоение знаний о взаимосвязи веществ различных классов (на примерах превращений неорганических и органических веществ);

Пример 11. Даны водные растворы: хлорида железа (III), иодоводорода, гексацианоферрат (II) калия и металлическое железо. Напишите уравнения четырех возможных реакций между этими веществами. 1) 2FeCl3 + 2HI = 2FeCl2 + I2 + 2HCl 2) 4FeCl3+ 3K4[Fe(CN)6] = Fe4[Fe(CN)6]3 ↓+12KCl3) 2FeCl3 + Fe = 3FeCl2

4) Fe + 2HI = FeI2+ H2↑

Пример 12.Напишите уравнения реакций, с помощью которых можно осуществить следующие превращения: Напишите уравнения реакций, с помощью которых можно осуществить следующие превращения:

C6H14   X1   X2   X3   X4   X3

Ответ:1) CH3 – (CH2)4 – CH3 C6H6 + 4H2

2) C6H6 + Cl2 C6H5Cl + HCl3) C6H5Cl + H2O C6H5OH + HCl4) C6H5OH + NaOH ® C6H5ONa + H2O5) C6H5ONa + CO2 + H2O ® C6H5OH + NaHCO3

– расчетные задачи.

Пример 13. В 71,4 мл 40%-ного раствора сульфида меди (II) (r = 1,12 г/мл) поместили кусочек натрия массой 2,3 г.Рассчитайте массовые доли веществ в образовавшемся растворе.

Ответ: 1) Составим уравнение реакции и рассчитаем количество вещества образовавшегося гидроксида натрия и водорода:2Na + H2O = 2NaOH + H2

n(Na) = 2,3/23 = 0,1 (моль)n(NaOH) = n(Na) = 0,1 (моль)n(H2) = ½n(Na) = 0,05 (моль)2) Составим уравнение реакции между растворами сульфата меди(II) и гидроксида натрия, рассчитаем количество вещества образовавшейся соли:CuSO4 + 2NaOH = Cu(OH)2¯ + Na2SO4

n(CuSO4) = 71,4 ´ 1,12 ´ 0,4/160 = 0,2 (моль)

16

Page 15: эл курс подг к егэ Word (2)

n(CuSO4) : n(NaOH) = 0,2 : 0,1 = 2 : 1CuSO4 – избытокn(Na2SO4) = ½n(NaOH) = 0,05 (моль)3) Рассчитаем массы веществ в образовавшемся растворе:m(Na2SO4) = 142 ´ 0,05 = 7,1 (г)m(CuSO4) = 160 ´ (0,2 – 0,05) = 24 (г)4) Рассчитаем массу образовавшегося раствора и массовые доли веществ в нем:m(р-ра) = m(Na) + m(р-ра CuSO4) – m(Cu(OH)2) – m(H2) == 2,3 + 80 – 98´0,05 – 2´0,05= 77,3 (г)w( CuSO4) = 24/77,3 = 0,31 или 31%w( Na2SO4) = 7,1/77,3 = 0,09 или 9%

Пример14.При взаимодействии 18 г неизвестной одноосновной карбоновой кислоты с избытком гидрокарбоната натрия выделилось 6,72л газа (н.у.). Установите молекулярную формулу кислоты.Ответ:1) Запишем уравнение реакции в общем виде и найдем количество вещества кислоты:СnH2n+1COOH + NaHCO3 → СnH2n+1COONa + CO2 + H2O n(CnH2n+1COOH) = n(CO2) = 6,72/22,4 = 0,3 (моль). 2) Рассчитаем молярную массу кислоты:М(CnH2n+1CООH) = 18/0,3 = 60 (г/моль)3) Определим число атомов углерода в молекуле кислоты и установим её формулу:М(CnH2n+1CООH) = 12n + 2n +1 + 12 + 16´2 + 1 = 14n + 4614n + 46 = 60n = 1CH3CООH

Занятие 2Входной контроль

Часть 1

А1. Электронную конфигурацию 1s22s22p63s23p63d5 частица

1) V0 2)Sc0 3) Fe2+ 4) Mn2+

17

Page 16: эл курс подг к егэ Word (2)

А2. Свойства химических элементов определяются

1) числом электронов в атоме 2) числом электронов на внешнем энергетическом уровне 3) числом нейтронов в атоме 4) зарядом атомного ядра

А3. Наибольшая энергия связи в молекуле

1) CI2 2) О2 3) N2 4) F2

А4. В соединениях: CO, HCOH, CH3OH углерод имеет степени окисления, соответственно равные

1) +2; +4; - 4; 2) +2; 0; -2; 3) -2; -4; +2; 4) -2; 0; +4;

А5. В узлах кристаллической решетки легкоплавких веществ находятся

1) ионы 2) атомы 3) атомы и ионы 4) молекулы

А6.Только амфотерные оксиды расположены в ряду

1) BeO, Cr2O3, FeO 2) Fe2O3, AI2O3, ZnO 3) ZnO, Cr2O3, Na2O 4) FeO, CO2, CuO

А7. Алюминий взаимодействует со всеми веществами ряда

1) CI2, HCI, SO2 2) HCI, KOH, CO2

3) H2SO4, NaOH, Br2

4) HNO3, Ca(OH)2, FeO

А8. Только восстановительные свойства проявляет

18

Page 17: эл курс подг к егэ Word (2)

1) азотистая кислота 2) сероводород 3) оксид серы (IV) 4) сера

А9. При обычных условиях с азотом реагирует

1) литий 2) натрий 3) железо 4)рубидий

А10. При взаимодействии оксида железа (III) с соляной кислотой образуется

1) FeCI2 и H2O 2) FeCI2 и H2

3) FeCI3 и H2O 4) FeCI3 и H2

А11. Верны ли следующие суждения о свойствах щелочей?

А. Щелочи термически неустойчивы. Б. Водные растворы щелочей изменяют окраску индикаторов.

1) верно только А 2) верно только Б 3) верны оба суждения 4) оба суждения неверны

А12. Хлорид меди (II) образуется при взаимодействии соляной кислоты с

1) медью 2) сульфатом меди (II) 3) иодидом меди (II) 4) оксидом меди (II)

А13. В схеме превращений CuS → X → CuO веществами «X» является

1) Cu(OH)2

2) CuCO3

3) Cu(NO3)2

4) Cu3(PO4)2

А14. Число изомеров состава C4H9CI равно

19

Page 18: эл курс подг к егэ Word (2)

1) 2 2) 3 3) 4 4) 5

А15. Верны ли следующие суждения о свойствах бензола? А. Все атомы углерода в бензоле находятся в состоянии sp2- гибридизации. Б. Реакции присоединения у бензола протекают труднее, чем у алкенов.

1) верно только А 2) верно только Б 3) верны оба суждения 4) оба суждения неверны

А16. При окислении пропанола-2 оксидом меди (II) образуется

1) ацетальдегид 2) пропионовая кислота 3) пропаналь 4) пропанон

А17. Олеиновая кислота не реагирует с

1) бромной водой 2) водородом 3) сульфатом натрия 4) этанолом

А18. В схеме превращений СН2 ═ СН2 → Х → С2Н5NH2

веществом «Х» является

1) этан 2) этанол 3) уксусная кислота 4) этин

А19. К реакциям присоединения и замещения относятся соответственно

1) CH3COOH + NaOH → и CuO + HCI 2) CH3COONa + NaOH и C6H5OH + Na→ 3) KBr + CI2 → и Fe + CuCI2

4) СН2 ═ СН2 + H2 и Zn + HCI →

А20. При повышении давления в 4 раза скорость реакции C(тв ) + 2CI2(г) = CCI4(г) возрастет в

20

Page 19: эл курс подг к егэ Word (2)

1) 4 раза 2) 8 раз 3) 16 раз 4) 64 раза

А21. Химическое равновесие в системе Fe3O4 (тв ) + 4CO(г) ⇄ 3Fe (тв ) +4 CO2(г)

_ Q смещается в сторону исходных веществ при

1) повышении давления 2) понижении давления 3) повышении температуры 4) понижении температуры

А22. При диссоциации ионы хлора не образует

1) NaCIO3

2) NH4CI 3) CaOHCl 4) CaCI2

А23. Сульфат натрия можно получить реакцией

1) NaCI + H2SO4(р-р ) → 2) NaCI + K2SO4 → 3) NaCI(тв) + H2SO4(конц ) → 4) NaOH + SO2 →

А24. Процессу окисления соответствует схема 1) MnO4

- → Mn+2

2) MnO4 - → Mn+4

3) Fe+2 → Fe+3 4) Fe+3 → Fe+2 А25. Фенолфталеин бесцветен в водном растворе

1) KOH 2) Na2SO3

3) HCI 4) Na2S

А26. Этилен можно получить взаимодействием хлорэтана с

1) водным раствором щелочи 2) спиртовым раствором щелочи

21

Page 20: эл курс подг к егэ Word (2)

3) водородом 4) серной кислотой (конц)

А27. Метановая кислота может реагировать с каждым веществом ряда

1) CuO и HNO3

2) Cu(OH)2 и [Ag(NH3)2]OH 3) CH3OH и CH3COOH 4) O2 и C6H5OH

А28. Белый осадок выпадает при взаимодействии бромной воды с

1) этиленом 2) этанолом 3) фенолом 4) бензолом

А29. Циркуляционным является процесс производства

1) чугуна 2) стали 3) серной кислоты 4) аммиака

А30. В результате реакции, термохимическое уравнение которой 2SO2 + O2 = 2SO3 +198 кДж выделилось 79,2 кДж тепла. Объём (н.у.) израсходованного при этом кислорода равен

1) 4,48 л 2) 6,72 л 3) 8,96 л 4) 13,44 л

22

Page 21: эл курс подг к егэ Word (2)

Часть 2

В1. Установите соответствие между названием вещества и его

принадлежностью к определенному классу (группе) неорганических

соединений.

НАЗВАНИЕ ВЕЩЕСТВА КЛАСС (ГРУППА) НЕОРГАНИЧЕСКИХ СОЕДИНЕНИЙ

А) сернистая кислота 1) амфотерный гидроксид Б) соляная кислота 2) щелочь В) гидроксид хрома (III) 3) кислородсодержащая кислотаГ) гидрокарбонат натрия 4) бескислородная кислота

5) средняя соль 6) кислая соль

В2. Установите соответствие между схемой химической реакции и

изменением степени окисления восстановителя.

СХЕМА РЕАКЦИИ ИЗМЕНЕНИЕ СТЕПЕНИ ОКИСЛЕНИЯ ОКИСЛИТЕЛЯА) I2 + CI2 + H2O ® HIO3+HCI 1) I- ® I0 Б) KI + KMnO4 +H2SO4 ® 2) CI+5® CI-

® MnSO4+I2 + K2SO4+ H2O 3) CI0® CI-

В) FeI2 + HNO3(р-р) ® Fe(NO3)3 + NO +I2 + H2O 4) I0 ® I+5

Г) NaCIO3 + NaI + HCI ® I2 + NaCI + H2O 5) Mn+7 ® Mn+2 6) N+5 ® N+2

В3. Установите соответствие между формулой вещества и процессом, который протекает на аноде в результате электролиза его водного

раствора.

ФОРМУЛА ВЕЩЕСТВА ПРОЦЕСС НА АНОДЕ А) NaOH 1) Na+ +1ē → Na0

Б) NaCI 2) Zn+2 +2ē → Zn+0

В) Na2SO4 3) 4OH– – 4ē → O2 + 2H2O Г) ZnSO4 4) 2CI– – 2ē → CI2

5) 2H2O – 4ē → O2 +4H+

6) 2H2O +2ē → H2 + 2OH–

В4. Установите соответствие между названием соли и типом гидролиза её

23

Page 22: эл курс подг к егэ Word (2)

водного раствора НАЗВАНИЕ СОЛИ ТИП ГИДРОЛИЗА А) сульфид алюминия 1) по аниону Б) нитрат железа (II) 2) по катиону В) сульфит натрия 3) по катиону и аниону Г) хлорид аммония

В5. И соляная кислота и гидроксид кальция способны реагировать с

1) AgNO3

2) Na2SiO3

3) SO2

4) BaCl2

5) FeSO4

6) Zn(OH)2

В6. Для этана характерны:

1) sp2-гибридизация орбиталей всех атомов углерода в молекуле 2) sp3-гибридизация орбиталей атомов углерода в молекуле 3) окисление раствором перманганата калия 4) реакция нитрования 5) реакция дегидрирования 6) взаимодействие с этанолом

В7. Реакция «серебряного зеркала» характерна для

1) уксусной кислоты 2) метановой кислоты 3) формальдегида 4) фенола 5) этилацетата 6) глюкозы

В8. С раствором гидроксида калия взаимодействуют :

1) метанол 2) метиламин 3) глицин 4) хлорид метиламмония 5) анилин 6) сульфат фениламмония

В9. К 60 г 25% раствора сульфата натрия добавили 40 мл воды. Массовая

24

Page 23: эл курс подг к егэ Word (2)

доля сульфата натрия в полученном растворе равна _________%. (Запишите число с точностью до целых.)

В10. Объём оксида углерода (IV), полученный при взаимодействии 50 л метана (н.у.) 40л кислорода, равен _________ л. (Запишите число с точностью до целых.)

Часть 3

С1. Используя метод электронного баланса, составьте уравнение реакции: K2Cr2O7 + HCI ® CrCI3 + …+ … + H2O Определите окислитель и восстановитель.

С2. Даны вещества: сульфат меди (II), иодид калия, гидрокарбонат натрия, гидроксид натрия. Напишите уравнения четырех возможных реакций между этими веществами

С3. Напишите уравнения реакций, с помощью которых можно осуществить следующие превращения: C2H2   X1   X2   X3   X4

 X5

С4. Оксид фосфора (V), полученный при окислении 3,1 г фосфора, поместили в 25 мл 25%-ного раствора гидроксида натрия (r=1,28г/мл). Определите состав образовавшейся соли и её массовую долю (в %) в растворе.

С5. При взаимодействии 18 г неизвестной одноосновной карбоновой кислоты с избытком гидрокарбоната натрия выделилось 6,72л газа (н.у.). Установите молекулярную формулу кислоты.

Как работать с пособием

1. Изучите тему по предложенному плану, используя учебники и учебные пособия из рекомендуемого списка.2. Выполните тестовые задания к данной теме.3. Сверьте свои ответы с правильными.4. Если при выполнении самостоятельной работы возникли затруднения или допущены ошибки, вернитесь к пункту 1.

Занятие 3

25

Page 24: эл курс подг к егэ Word (2)

СТРОЕНИЕ АТОМА

План изучения темы1. Атом, его состав, характеристика элементарных частиц.2. Изотопы как разновидности атомов элементов.3. Строение электронных оболочек атомов первых четырех периодов: s-, p- и d- элементы. Электронная конфигурация атома. Основное и возбужденное состояние атомов.

Тест для самостоятельной работы

1. Атом металла, высший оксид которого Ме2О3, имеет электронную формулу внешнего энергетического уровня

1) ns2 np1 2) ns2 np2 3) ns2 np3 4) ns2 np5

2. Атому неона соответствует электронная конфигурация частицы

1) P 2) Cl- 3) K 4) Na+

3. Число нейтронов в ядре атома 40К равно

1) 19 2) 20 3) 21 4) 59

4. В атоме фосфора валентными являются орбитали подуровней

1) 3s, 3p и 3d 2) только 3s и 3p 3) только 3p 4) 2p, 3s и 3p

5. Электронная конфигурация 1s2 2s2 2p6 3s2 3p6 соответствует иону

1) Zn 2+ 2) Al 3+ 3) Cr3+ 4) Ca2+

6. Электронную формулу 1s22s22p63s23p64s2 имеет атом элемента

1) Ba 2) Mg 3) Ca 4) Sr

7. Наименьшее число протонов содержится в ядре атома

1) кислорода 2) кальция 3) алюминия 4) хлора

8. Атомы элементов VA группы имеют электронную конфигурацию внешнего энергетического уровня

1) ns2 np1 2) ns2 np2 3) ns2 np3 4) ns2 np5

26

Page 25: эл курс подг к егэ Word (2)

9. Изотопы одного и того же элемента отличаются друг от друга

1) числом нейтронов 2) числом электронов 3) числом протонов 4) зарядом ядра10. Ион, в составе которого 14 протонов 18 электронов, имеет заряд

1) 4+ 2) 2- 3) 2+ 4) 4-

11. Сумма протонов, нейтронов и электронов в атоме 40Са равна

1) 40 2) 60 3) 30 4) 50

12. Восьмиэлектронную внешнюю оболочку имеет ион

1) S4+ 2) S2- 3) Br5+ 4) Sn2+

13. Внешний энергетический уровень атома элемента, образующего высший оксид состава ЭО3, имеет формулу

1) ns2np1 2) ns2np2 3) ns2np3 4) ns2np4

14. Электронная формула катиона Mg2+

1) 1s2 2s2 2p4

2) 1s2 2s2 2p6

3) 1s2 2s2 2p6 3s2

4) 1s2 2s2 2p6 3s2 3p2

15. Электронная формула аниона Cl- 1) 1s2 2s2 2p6 3s2 3p3

2) 1s2 2s2 2p6 3s2 3p4

3) 1s2 2s2 2p6 3s2 3p5

4) 1s2 2s2 2p6 3s2 3p6

16. Электронную конфигурацию 1s22s22p63s23p64s2 в основном состоянии имеет атом

1) лития 2) натрия 3) калия 4) кальция

17. Электронная конфигурация внешнего электронного слоя атома серы в невозбужденном состоянии

1) 4s2 4p6 2) 3s2 3p6 3) 3s2 3p4 4) 4s2 4p4

27

Page 26: эл курс подг к егэ Word (2)

18. Число электронов в ионе Mg2+ равно

1) 12 2) 10 3) 8 4) 11

19. Среди элементов VА группы максимальный радиус атома имеет

1) азот 2) фосфор 3) сурьма 4) висмут

20. Одинаковую конфигурацию внешнего энергетического уровня имеют атомы лития и 1) бериллия 2) меди 3) рубидия 4) магния

21. У атома какого элемента в основном состоянии имеется два неспаренных электрона?

1) кремния 2) азота 3) бора 4) хлора

22.Число электронов в ионе Cu1+ равно

1) 29 2) 64 3) 27 4) 28

23. Количество электронов в атоме определяется

1) числом протонов 2) числом нейтронов 3) числом энергетических уровней 4) величиной относительной атомной массы

24. Химический элемент, формула высшего оксида которого R2O3, имеет следующую электронную конфигурацию атома:

1) 1s2 2s2 2p6 3s1

2) 1s2 2s2 2p6 3s2 3p5

3) 1s2 2s2 2p6 3s2 3p6 4s1

4) 1s2 2s2 2p1

25. В каком ряду химические элементы расположены в порядке возрастания их атомного радиуса?

1) Li, Be, B, C 2) P, S, Cl, Ar

28

Page 27: эл курс подг к егэ Word (2)

3) Sb, As, P, N 4) F, Cl, Br, I

26. Атом элемента, максимальная степень окисления которого +4, в основном состоянии имеет электронную конфигурацию внешнего энергетического уровня:

1) 3s23p4 2) 2s22p2 3) 2s22p4 4) 2s22p6

27. На третьем энергетическом уровне имеется по 8 электронов у каждой из частиц:

1) Na+ и Ar 2) S2- и Ar 3) F - и N e 4) Mg 2+ и S

28. Шесть валентных электронов на внешнем энергетическом уровне содержит атом

1) фосфора 2) серы 3) хрома 4) кремния

29. Наибольшее значение электроотрицательности имеет атом, электронная конфигурация которого

1) 1s2 2s2 2p6 2) 1s2 2s2 2p4 3) 1s2 2s2 2p5 4) 1s2 2s2 2p6 3s2 3p5

30. Элемент, имеющий электронную конфигурацию 1s2 2s2 2p6 3s2 3p6 3d5 4s2 расположен в

1) 3 периоде, IIА группе2) 4 периоде, IIА группе3) 4 периоде, VIIБ группе4) 4 периоде, IVА группе

Занятие 4 Периодический закон и периодическая система химических

элементов Д. И. Менделеева.

План изучения темы

29

Page 28: эл курс подг к егэ Word (2)

1) Периодический закон и периодическая система химических элементов Д. И. Менделеева.

2) Группы, подгруппы, периода, ряды.3) Знаки химических элементов, их названия.4) Радиусы атомов, их периодические изменения в системе

химических элементов.5) Классификация химических элементов.6) Закономерности изменения химических свойств элементов и их

соединений по периодам и группам.7) Значение периодического закона.

Тест для самостоятельной работы

1. А-группы периодической системы Д. И. Менделеева образованы

1) только s- элементами 2) только p-элементами 3) s- и p- элементами 4) s- и d- элементами

2. В ряду химических элементов Si → P → S → Cl неметаллические свойства

1) ослабевают2) усиливаются3) не изменяются

4) изменяются периодически

3. К семейству s- элементов относится

1) водород 2) кислород 3) фтор 4) аргон

4. По увеличению неметаллических свойств химические элементы расположены в ряду:

1) Al, Mg, Na 2) C, Si, Ge 3) Li, Na, K 4) N, O, F

5. Номер группы в периодической системе Д. И. Менделеева соответствует

1) числу энергетических уровней в атоме 2) числу валентных электронов в атоме 3) числу неспаренных электронов в атоме

30

Page 29: эл курс подг к егэ Word (2)

4) общему числу электронов в атоме

6. Наименее электроотрицательный химический элемент

1) железо 2) магний 3) азот 4) кальций

7. В ряду Al → Si → P → S

1) увеличивается число электронных слоев в атомах 2) усиливаются неметаллические свойства 3) уменьшается число протонов в ядрах атомов 4) возрастают радиусы атомов

8. В ряду элементов Na → Mg → Al → Si

1) увеличивается число электронных слоев в атомах 2) усиливаются металлические свойства 3) уменьшается высшая степень окисления элементов 4) ослабевают восстановительные свойства элементов

9. Кислотные свойства водородных соединений элементов VI-А группы с увеличением порядкового номера

1) усиливаются 2) ослабевают 3) остаются неизменными 4) изменяются периодически10. Кислотные свойства оксидов в ряду SiO2 → P2O5 → SO3

1) ослабевают 2) усиливаются 3) не изменяются 4) изменяются периодически

11. Какой элемент образует газообразное водородное соединение, соответствующее общей формуле RH2?

1) бор 2) калий 3) сера 4) хром

12. У какого элемента наиболее выражены неметаллические свойства?

1) Si 2) C 3) Ge 4) Sn

31

Page 30: эл курс подг к егэ Word (2)

13. Оснóвные свойства оксидов элементов II-А группы с увеличением порядкового номера

1) уменьшаются 2) возрастают 3) не изменяются 4) изменяются периодически

14. У какого элемента наиболее выражены металлические свойства?

1) Na 2) Ca 3) Mg 4) K

15. По периоду слева направо возрастают

1) основные свойства гидроксидов 2) восстановительные свойства водородных соединений 3) кислотные свойства гидроксидов 4) оснóвные свойства оксидов

16. У элементов подгруппы углерода с увеличением атомного номера уменьшается

1) атомный радиус 2) заряд ядра атома 3) число валентных электронов в атомах 4) электроотрицательность

17. Устойчивость водородных соединений элементов VIА группы с ростом заряда ядра атома

1) увеличивается 2) не изменяется 3) ослабевает 4) изменяется периодически

18. В главных подгруппах периодической системы сверху вниз оснóвные свойства гидроксидов металлов

1) увеличиваются 2) уменьшаются 3) не изменяются 4) изменяются периодически

32

Page 31: эл курс подг к егэ Word (2)

19. В главных подгруппах периодической системы сверху вниз электроотрицательность элементов

1) уменьшается 2) увеличивается 3) не изменяется 4) изменяется периодически

20. Высший оксид с оснóвными свойствами образует

1) литий 2) бор 3) углерод 4) азот

21. В ряду элементов азот – кислород – фтор возрастает

1) валентность 2) число энергетических уровней3) число внешних электронов4) число неспаренных электронов

22. В ряду химических элементов: бор – углерод – азот возрастает

1) способность атомов отдавать электроны2) высшая степень окисления3) низшая степень окисления4) радиус атома

23. В порядке возрастания атомного радиуса химические элементы расположены в ряду:

1) Na, Ca, Al, Si 2) Li, Na, K, Rb 3) P, S, Cl, Ar 4) N, O, F, N

24. С ростом заряда ядра атомов кислотные свойства оксидов в ряду N2O5 → P2O5 → As2O5 → Sb2O5

1) ослабевают 2) усиливаются 3) не изменяются 4) изменяются периодически

25. Строение атомов элементов второго периода сходно по

33

Page 32: эл курс подг к егэ Word (2)

1) числу электронов в атомах 2) числу электронов на внешнем слое 3) числу заполненных энергетических уровней 4) распределению электронов по орбиталям

26. В ряду оксидов: Na2O – MgO – Al2O3 – SiO2

оснóвные свойства

1) уменьшаются2) усиливаются3) не изменяются4) сначала уменьшаются, потом увеличиваются

27. Кислотный характер наиболее выражен у высшего оксида, образованного

1) бериллием 2) бором 3) фосфором 4) кремнием

28. Атомы химических элементов второго периода имеют одинаковое число

1) электронов на внешнем уровне 2) нейтронов 3) заполненных энергетических уровней 4) протонов

29. Неметаллические свойства элементов усиливаются в ряду:

1) Cl – S – P – Si 2) N – P – As – Se 3) B – C – N – O 4) C – Si – P – N

30. В каком ряду химические элементы расположены в порядке уменьшения их атомного радиуса?

1) Li, Na, K, Rb 2) C, Si, Ge, Sn 3) B, Al, Ga, In 4) Sr ,Ca, Mg, Be

Занятие 5

34

Page 33: эл курс подг к егэ Word (2)

Химическая связь и кристаллические решетки

План изучения темы1.Ковалентная связь, механизмы ее образования2. Полярная и неполярная ковалентная связь.3.Свойства ковалентной связи: длина, энергия, гибридизация.4. Ионная связь, механизмы ее образования. Свойства ионной связи.5. Металлическая связь.6. Водородная связь.7. Вещества молекулярного и немолекулярного строения.8. Кристаллические решетки: атомные, молекулярные, ионные и металлические. Зависимость свойств веществ от особенностей их кристаллической решетки.

Тест для самостоятельной работы

1. По донорно-акцепторному механизму образована одна из ковалентных связей в частице

1) NH4+ 2) H4 3) SiH4 4) CBr4

2. Наибольшая полярность связи – в молекуле

1) сероводорода 2) хлора 3) фосфина 4) хлороводорода

3. В сероуглероде CS2 химическая связь

1) ионная 2) металлическая 3) ковалентная полярная 4) ковалентная неполярная

4. Соединением с ковалентной неполярной связью является

1) HCl 2) O2 3) CaCl2 4) H2O

5. Ионный характер связи наиболее выражен в соединении

1) CCl4 2) SiO2 3) CaBr2 4) NH3

6. Вещество с ковалентной неполярной связью имеет формулу

1) NH3 2) I2 3) H2S 4) CF4

35

Page 34: эл курс подг к егэ Word (2)

7. Веществом с неполярной ковалентной связью является

1) белый фосфор 2) оксид фосфора (V) 3) оксид углерода (II) 4) оксид кремния (IV)

8. Вещество, обладающее электронной проводимостью, ковкостью, блеском, образовано химической связью

1) атомной 2) ионной 3) металлической 4) молекулярной

9. Ковалентная неполярная связь образуется между 1) атомами металла и неметалла 2) ионами металла и неметалла 3) атомами одного и того же неметалла 4) атомами различных неметаллов

10. Химическая связь в хлороводороде и хлориде бария соответственно 1) ковалентная полярная и ионная 2) ковалентная неполярная и ионная 3) ковалентная полярная и металлическая 4) ковалентная неполярная и металлическая11. В аммиаке и фториде лития химическая связь соответственно

1) ионная и ковалентная полярная 2) ковалентная полярная и ионная 3) ковалентная неполярная и металлическая 4) ковалентная неполярная и ионная

12. В каком ряду записаны формулы веществ только с ковалентной полярной связью? 1) Cl2, PH3, HCl 2) HBr, NO, Br2

3) H2S, H2O, S8

4) HI, H2O, NH3

13. Веществом с ковалентной полярной связью является

1) Cl2 2) NaBr 3) H2S 4) MgCl2

14. Вещество с ковалентной неполярной связью имеет формулу

1) NH3 2) CH4 3) H2S 4) F2

36

Page 35: эл курс подг к егэ Word (2)

15. Вещества только с ионной связью приведены в ряду:

1) F2, CCl4, KCl 2) NaBr, Na2O, KI 3) SO2, P4, CaF2

4) H2S, Br2, K2S

16. Соединение с ионной связью образуется при взаимодействии

1) CH4 и O2

2) SO3 и H2O 3) C2H6 и HNO3

4) NH3 и HCl

17. Формула вещества с ковалентной связью

1) HCl 2) NaCl 3) LiF 4)KI

18. Вещество с ковалентной полярной связью имеет формулу

1) KF 2) SiH4 3) F2 4) CaF2

19. Ковалентные связи имеет каждое из веществ, указанных в ряду:

1) C3H8, NO2, NaF 2) KCl, CH3Cl, C6H12O6

3) P2O5, NaHSO3, Ba 4) C6H5NH2, P4, CH3OH

20. Ковалентную связь имеет каждое из веществ, указанных в ряду

1) CaO, C3H6, S8

2) Fe, NaNO3, CO 3) N2, CuCO3, K2S 4) C6H5NO2, SiO2, CHCl3

21. Водородная связь характерна для

1) алканов 2) альдегидов 3) спиртов 4) алкинов

22. Хлорид калия имеет кристаллическую решетку

1) атомную 2) молекулярную

37

Page 36: эл курс подг к егэ Word (2)

3) ионную 4) металлическую

23. Для веществ с атомной кристаллической решеткой характерна

1) высокая твердость 2) низкая температура плавления 3) низкая температура кипения 4) летучесть

24. Какие из утверждений являются верными? А. Вещества с молекулярной решеткой имеют низкие температуры плавления и низкую электропроводность. Б. Вещества с атомной решеткой пластичны и обладают высокой электрической проводимостью.

1) верно только А2) верно только Б3) верны оба утверждения4) оба утверждения неверны

25. Для веществ с ионной кристаллической решеткой характерна

1) низкая температура плавления 2) низкая электропроводность 3) низкая температура кипения 4) высокая температура плавления

26. Наименьшую температуру плавления имеет

1) F2 2) Cl2 3) Br2 4) I2

27. Для летучего, легкоплавкого вещества, обладающего низкой теплопроводностью, характерна кристаллическая решетка

1) атомная 2) ионная 3) молекулярная 4) металлическая

28. Молекулярное строение имеет вещество, формула которого

1) CO2 2) KBr 3) MgSO4 4)Na2S

29. Молекулярное строение имеет

1) KOH 2) Cu (NO3)2 3) C3H7OH 4) ZnSO4

38

Page 37: эл курс подг к егэ Word (2)

30. Немолекулярное строение имеет каждое из двух веществ:

1) S8 и O2 2) Fe и NaCl 3) CO и Mg 4) Na2CO3 и I2 (тв)

Занятие 6

Валентность и степень окисления

План изучения темы

1. Валентные возможности атомов химических элементов.2. Степень окисления элементов.3. Правила определения степеней окисления элементов.4. Степени окисления: низшая, высшая, промежуточная.5. Электроотрицательность.

Тест для самостоятельной работы

1. Элементом, у которого высшая степень окисления равна +7, является

1) O 2) P 3) Si 4) Cl

2. Степень окисления –1 кислород имеет в соединении

1) H2O 2) N2O5 3) NaOH 4) Na2O2

3. Атом хлора имеет степень окисления +5 в соединении

1) HCl 2) Cl2O7 3) CaCl2 4) KClO3

4. Степень окисления + 5 азот имеет в соединении

1) HNO2 2) N2O3 3) NaNO3 4) NH4NO2

5. Степень окисления атома азота в сульфате аммония равна

1) +5 2) +3 3) – 3 4) +4

6. Степень окисления + 3 азот проявляет в каждом из двух соединений:

1) HNO2 и NH3

2) NH4Cl и N2O3

3) NaNO2 и NF3

4) HNO3 и N2

7. В каком ряду указаны соединения с одинаковой степенью окисления селена? 1) SeO2, K2SeO3, SeF4

2) H2Se, SeO2, K2SeO3

39

Page 38: эл курс подг к егэ Word (2)

3) K2Se, H2Se, H2SeO4

4) SeO3, K2SeO3, SeO2

8. Высшую степень окисления хлор проявляет в соединении 1) HCl 2) HClO 3) KClO3 4) HClO4

9. Одинаковую степень окисления азот имеет а соединениях 1) NH4Cl, NH3

2) KNO2, NO2

3) NH4Cl, NO2

4) HNO3, N2O3

10. В порядке возрастания электроотрицательности элементы расположены в ряду:

1) S, Cl, F 2) S, F, O 3) C, Si, Ge 4) Si, Al, Mg

11. Максимальную степень окисления марганец проявляет в соединении

1) KMnO4 2) MnO2 3) K2MnO4 4) MnSO4

12. Степень окисления – 3 фосфор проявляет в соединении

1) PH3 2) P2O3 3) NaH2PO4 4) H3PO4

13. Наименьшую степень окисления сера проявляет в соединении

1) Na2S 2) Na2SO3 3) Na2SO4 4) SO3

14. Наибольшую степень окисления марганец проявляет в соединении

1) MnCl2 2) MnO 3) K2MnO4 4) Mn2O7

15. Степень окисления – 3 азот проявляет в каждом из двух соединений:

1) HNO3 и NH3

2) NH3 и N2O3

3) NH3 и (NH4)2SO4

4) N2O3 и HNO2

16. Электроотрицательность химических элементов увеличивается в ряду:

1) Be, Mg, Ca 2) F, Cl, Br 3) P, S, Cl 4) Cl, S, P

17. В порядке возрастания электроотрицательности элементы расположены в ряду

1) H – S – Cl – O – F 2) F – O – Cl – S – H 3) H – Cl – S – O – F

40

Page 39: эл курс подг к егэ Word (2)

4) H – S – Cl – F – O

18. В ионе аммония РH4+ степень окисления фосфора равна

1) + 1 2) + 3 3) – 3 4) – 5

19. Степень окисления, равную – 2, азот имеет в соединении

1) NH3 2) NH2OH 3) N2H4 4) N2O4

20. Наименее электроотрицательный элемент –

1) хром 2) магний 3) сера 4) кальций

21. Степень окисления фосфора в NaH2PO4 равна

1) – 3 2) 0 3) + 3 4) +5

22. Степень окисления серы в NaHSO3 равна

1) + 6 2) –2 3) 0 4) + 4

23. Одинаковую степень окисления азот проявяляет в веществах, указанных в ряду:

1) N2O5, HNO3, NaNO3

2) NO2, HNO2, KNO3

3) NO, NO2, N2O3 4) HNO3, HNO2, NO2

24. Наименьшую степень окисления сера проявляет в соединении

1) Na2SO4 2) H2S 3) SO2 4) K2SO3

25. Степень окисления -3 азот проявляет в соединении

1) (NH4)2CO3

2) N2O4

3) HNO2

4) Al(NO3)3

26. Наибольшей электроотрицательностью среди элементов VА группы обладает 1) фосфор 2) сурьма 3) азот 4) мышьяк

41

Page 40: эл курс подг к егэ Word (2)

27. В молекуле какого вещества углерод проявляет низшую степень окисления?

1) углекислый газ2) метан3) угарный газ4) угольная кислота

28. Отметьте формулу вещества, в котором степень окисления углерода равна -4:

1) СО2

2) СН4

3) НСООН 4) СО

29. В каком случае правильно указана максимальная степень окисления элемента?

1) H+ 2) S4+ 3) Cl5+ 4) Fe3+

30. Степень окисления хрома в соединении К2Сr2О7 равна: 1) +3 2) +6 3) +7 4) –2

Занятие 7Химические реакции

План изучения темы

1. Физические и химические явления.2. Типы химических реакций.3. Окислительно- восстановительные реакции.4. Признаки химических реакций.

Тест для самостоятельной работы1. Взаимодействие хлорида меди (II) с железом относится к реакциям 1) разложения 2) обмена 3) замещения 4) соединения 2. Взаимодействие кислорода и азота относится к реакциям

1) разложения, эндотермическим2) обмена, экзотермическим3) соединения, эндотермическим4) соединения, экзотермическим

42

Page 41: эл курс подг к егэ Word (2)

3. Реакции, в ходе которых происходит взаимодействие ионов водорода с гидроксид - ионами, относят к реакциям

1) нейтрализации 2) соединения 3) замещения 4) разложения

4. Взаимодействие карбоната натрия с соляной кислотой относят к реакциям

1) разложения 2) обмена 3) замещения 4) соединения

5. Разбавленная серная кислота вступает в реакцию замещения с

1) гидроксидом натрия2) железом3) оксидом меди (II)4) серебром

6. Взаимодействие хлора с железом относится к реакциям

1) разложения, эндотермическим2) обмена, экзотермическим3) соединения, эндотермическим4) соединения, экзотермическим

7. К реакциям замещения и разложения относятся соответственно:

1) Al2O3 + 6HCl = 2AlCl3 + 3H2O и 2KClO3 = 2KCl + 3O2

2) C2H4 + H2O → C2H5OH и CuCl2 + Zn = ZnCl2 + Cu 3) Mg + 2HCl = MgCl2 + H2 ↑ и Cu(OH)2 = CuO + H2O 4) 2NaOH + H2SO4 = Na2SO4 + 2H2O и C2H6 → 2C + 3H2↑

8. Из перечисленных типов реакций всегда бывают только окислительно- восстановительными:

1) соединения 2) разложения 3) замещения 4) обмена 9. Реакция синтеза аммиака N2(г) + 3H2(г) ⇄ 2NH3(г) + Q

1) обратимая, некаталитическая, экзотермическая2) обратимая, каталитическая, эндотермическая3) обратимая, каталитическая, экзотермическая4) необратимая, каталитическая, экзотермическая

10. При нагревании сахарозы с водой в присутствии кислоты происходит

43

Page 42: эл курс подг к егэ Word (2)

реакция

1) гидратации2) этерификации3) гидролиза4) нейтрализации

11. Коэффициент перед формулой восстановителя в уравнении реакции между железом и хлором равен

1) 1 2) 2 3) 3 4) 4

12. Взаимодействие водорода с хлором относится к реакциям

1) разложения, эндотермическим2) обмена, экзотермическим3) соединения, эндотермическим4) соединения, экзотермическим

13. Бромирование пропилена относится к реакциям

1) присоединения2) замещения3) гидрогенезации4) изомеризации

14. Реакциями присоединения и замещения соответственно являются: 1) C3H8 + Cl2 и C6H6 + Cl2 2) C2H2 + HBr → и C6H6 + Cl2 3) C2H4 + Br2 → и Zn + HCl → 4) C6H5OH + HNO3→ и ZnO + HCl →

15. В реакции оксида меди (II) с водородом окислителем является

1) Cu2+ 2) H20 3) Cu0 4) H+

16. Реакция полимеризации не характерна для

1) бутадиена2) ацетилена3) этана4) винилхлорида

17. Взаимодействие сульфата меди (II) с гидроксидом натрия относится к

44

Page 43: эл курс подг к егэ Word (2)

реакции

1) замещения2) обмена3) разложения4) соединения

18. Реакциями обмена и замещения соответственно являются: 1) CH3COOH + Cl2 и HCOOH + Na2CO3 → 2) CH3COOH + NaOH → и H2О + Na2O → 3) CuO + H2SO4 → и CH3COOH + Na → 4) C2H6 + Br2 → и CuO + H2 → 19. Реакцией нейтрализации является реакция, схема которой

1) CaCO3 + HCl → CaCl2 + H2O + CO2

2) NaOH + HCl → NaCl + H2O3) BaCl2 + Na2SO4 → BaSO4↓ + NaCl4) FeCl2 + NaOH → Fe(OH)2↓ + NaCl

20. К реакциям замещения относится взаимодействие

1) этена и воды2) брома и водорода3) брома и пропана4) метана и кислорода

21. Азот является восстановителем в реакции

1) N2 + O2 → 2) N2 + H2 → 3) N2 + Mg → 4) N2 + C →

22. Только окислительные свойства проявляет

1) сульфид натрия2) сера3) серная кислота4) сульфит калия

23. Согласно схеме N+5 + nē → N–3

число принятых электронов (n) равно 1) 5 2) 2 3) 3 4) 8

24. Элементом, для которого возможно изменение степени окисления

45

Page 44: эл курс подг к егэ Word (2)

Э–3 → Э0, является

1) N 2) Cl 3) S 4) Al

25. В реакции цинка с разбавленной серной кислотой восстановителем является 1) Zn0 2) SO4

2– 3) H+ 4)Zn2+

26. Схеме превращения N–3 → N0

соответствует уравнение реакции

1) N2 + H2 → NH3

2) N2 + O2 → 2NO3) 4NH3 + 5O2 → 4NO + 6H2O4) 4NH3 + 3O2 → 2N2 + 6H2O

27. Процессу восстановления соответствует схема

1) NH3 → NH4+

2) NO3– → NH3

3) Cr → Cr3+

4) CH4 → CO2

28. Аммиак является восстановителем в реакции с

1) водой2) оксидом меди (II)3) азотной кислотой4) хлороводородом

29. Оксид серы (IV) в химических реакциях

1) не проявляет окислительно-восстановительных свойств2) является только окислителем3) является только восстановителем4) может быть как окислителем, так и восстановителем

30. Реакции уравнение которой 4NH3 + 5O2 4NO + 6H2Ocоответствует схема превращения азота 1) N+3 → N+2 2) N–3 → N–2 3) N+3 → N–3 4) N–3 → N+2

Занятие 8

Тепловой эффект химической реакции

46

Page 45: эл курс подг к егэ Word (2)

План изучения темы

1. Направления течения реакций.2. Тепловые эффекты реакций. Реакции экзо- и эндотермические.

Тест для самостоятельной работы

1. В соответствии с термохимическим уравнением CH4 (г) + 2O2 (г) = CO2 (г) + 2H2O(г) + 802 кДж, при образовании 2 моль углекислого газа

1) выделяется 1604 кДж теплоты2) поглощается 1604 кДж теплоты3) выделяется 802 кДж теплоты4) поглощается 802 кДж теплоты

2. В соответствии с термохимическим уравнением 3Fe3O4 (к) + 8Al (к) = 9Fe(к) + 4Al2O3(к) + 3330 кДж,при образовании 18 моль железа

1) выделяется 3330 кДж теплоты2) выделяется 6660 кДж теплоты3) поглощается 3330 кДж теплоты4) поглощается 6660 кДж теплоты

3. В соответствии с термохимическим уравнением 2Na (к) + Cl2 (г) = 2NaCl (к) + 411,3 кДж,при взаимодействии 3 моль хлора с избытком натрия1) выделяется 411,3 кДж теплоты2) поглощается 411,3 кДж теплоты3) выделяется 1233,9 кДж теплоты4) поглощается 1233,9 кДж теплоты

4. В соответствии с термохимическим уравнением C6H12O6 (к) + 6O2 (г) ⇄ 6CO2 (г) + 6H2O (ж) +2800 кДж, при образовании 12 моль углекислого газа

1) выделяется 2880 кДж теплоты2) поглощается 2800 кДж теплоты3) выделяется 5600 кДж теплоты4) поглощается 5600 кДж теплоты

5. В результате реакции, термохимическое уравнение которой 2KCO3 (тв) = 2KCl (тв) + 3O2 (г) + 91 кДж, выделилось 182 кДж теплоты. Масса образовавшегося при этом кислорода равна

47

Page 46: эл курс подг к егэ Word (2)

1) 96 г 2) 192 г 3) 288 г 4) 576 г

6. В результате реакции, термохимическое уравнение которой 4FeS2 (тв) + 11O2(г) = 8SO2 (г) + 2Fe2O3(тв) + 3310 кДж, выделилось 6620 кДж теплоты. Масса образовавшегося оксида железа (III) равна

1) 320 г 2) 160 г 3) 480 г 4) 640 г

7. В результате реакции, термохимическое уравнение которой 2AgNO3 (тв) = 2Ag (тв) + 2NO2(г) + O2(г) – 317 кДж, поглотилось 1, 585 кДж теплоты. Масса разложившегося нитрата серебра составляет

1) 3,4 г 2) 1, 7 г 3) 10,2 г 4) 1.12 г

8. Теплота образования 1 моль оксида алюминия равна 1676 кДж/моль. Определите тепловой эффект реакции, в которой при взаимодействии алюминия с кислородом получено 25,5 г Al2O3.

1) 140 кДж 2) 209,5 кДж 3) 419 кДж 4) 838 кДж

9. Теплота образования 1 моль оксида магния из простых веществ составляет 600 кДж. Объем кислорода (н. у.), затраченный для получения 2400 кДж теплоты, равен

1) 11,2 л 2) 22,4 л 3) 33,6 л 4) 44,8 л

10. В результате реакции, термохимическое уравнение которой MgCO3 = MgO + CO2 – 102 кДж выделилось 4,48 л углекислого газа. Количество затраченной при этом теплоты составило

1) 204 кДж 2) 102 кДж 3) 1,02 кДж 4) 20,4 кДж

11. Теплота образования 1 моль хлороводорода из простых веществ составляет 92,3 кДж. Количество теплоты, выделяемое при образовании 4 моль HCl, равно

1) 184,6 кДж 2) 276,9 кДж 3) 369,2 кДж 4) 461,5 кДж12. Для получения 25 г железа согласно уравнению Fe2O3(тв) + 3CO(г) = 2Fe(тв) + 3CO2(г) – 27 кДж потребуется затратить теплоты

48

Page 47: эл курс подг к егэ Word (2)

1) 4,8 кДж 2) 6 кДж 3) 12 кДж 4)13,5 кДж

13. В результате реакции, термохимическое уравнение которой CuS2 (тв) + 2O2(г) = 2CuO(тв) + SO2(г) + 530 кДж, выделилось 795 кДж теплоты. Масса образовавшегося при этом оксида меди (II) равна

1) 30 г 2) 240 г 3) 60 г 4) 90 г

14. Согласно термохимическому уравнению реакции 4K2Cr2O7 (т) = 4K2CrO4 (т) + 2Cr2O3(т) + 3O2(г) – 398 кДж, для получения 6,72 л кислорода (н.у.) потребуется теплота количеством

1) 79,6 кДж 2) 59,4 кДж 3) 39,8 кДж 4) 29,7 кДж

15. Теплота образования 1 моль жидкой воды составляет 286 кДж. При взаимодействии 4 моль водорода с кислородом выделится теплота количеством

1) 572 кДж 2) 715 кДж 3) 858 кДж 4) 1144 кДж

16. В соответствии с термохимическим уравнением реакции 4P + 5O2 = 2P2O5 + 3010 кДж 150,5 кДж теплоты выделится, если на горение фосфора израсходуется кислород, объемом (н.у.) 1) 112 л 2) 11,2 л 3) 5,6 л 4) 224 л

17. В соответствии с термохимическим уравнением реакции C + O2 = CO2 + 393, 3 кДж при выделении 1180, 9 кДж теплоты масса сгоревшего углерода равна

1) 12 г 2) 24 г 3) 36 г 4) 72 г

18. В соответствии с термохимическим уравнением CH4 + 2O2 = CO2 + 2H2O + 896 кДж, масса метана, необходимая для выделения 1792 кДж теплоты, равна

1) 16 г 2) 32 г 3) 48 г 4) 64 г

19. Согласно термохимическому уравнению реакции 2KClO3(т) = 2KCl(т) + 3O2(г) + 78 кДж при образовании 112 л кислорода (н.у.) количество выделившееся теплоты будет равно

49

Page 48: эл курс подг к егэ Word (2)

1) 130 кДж 2) 65 кДж 3) 43,3 кДж 4) 13,0 кДж

20. В результате реакции, термохимическое уравнение которой C6H12O6 + 6O2 = 6CO2 + 6H2O + 2800 кДж, выделилось 1120 кДж теплоты. Количество вещества кислорода, вступившего в реакцию, равно

1) 1,2 моль 2) 9 моль 3) 3 моль 4) 2.4 моль

21. В соответствии с термохимическим уравнением 2HgO = 2Hg + O2 – 180 кДж, для разложения 2,17 г оксида ртути необходима теплота количеством

1) 1,8 кДж2) 0,45 кДж3) 0,9 кДж4) 9 кДж

22. В соответствии с термохимическим уравнением CH4 + 2O2 = CO2 + 2H2O + 896 кДж,

89,6 кДж теплоты выделится при горении метана количеством вещества

1) 0,1 моль 2) 0,2 моль 3) 0,25 моль 4) 0,5 моль

23. При окислении 4,8 г угля до оксида углерода (II) согласно уравнению 2C(тв) + O2(г) = 2CO(г) + 220 кДж выделится теплота количеством

1) 22 кДж 2) 44 кДж 3) 88 кДж 4) 220 кДж

24. При сжигании оксида углерода (II) согласно уравнению реакции 2CO(г) + O2(г) = 2CO2(г) + 566 кДж выделилось 152 кДж теплоты. Объем (н.у.) сгоревшего газа составил

1) 6 л 2) 12 л 3) 44,8 л 4) 120 л

25. В результате реакции, термохимическое уравнение котрой 4FeS2 (тв) + 11O2(г) = 8SO2 (г) + 2Fe2O3(тв) + 3310 кДж, выделилось 4965 кДж теплоты. Масса вступившего в реакцию FeS2

равна

50

Page 49: эл курс подг к егэ Word (2)

1) 720 г 2)180 г 3) 360 г 4) 520 г

26. В соответствии с термохимическим уравнением CH4 + 2O2 = CO2 + 2H2O + 896 кДж,

объем метана, необходимый для выделения 1782 кДж теплоты, равен 1) 11,2 л 2) 22,4 л 3) 44,8 л 4) 67,2л

27. В соответствии с термохимическим уравнением C(тв) + O2(г) = CO2(г) +394 кДж для получения 422,2 кДж теплоты следует затратить кислород объемом (н.у.)

1) 20 л 2) 22 л 3) 24 л 4) 26 л

28. В результате реакции, термохимическое уравнение которой C6H12O6(г) + 6O2(г) = 6CO2(г) + 6H2O + 2800 кДж выделилось 700 кДж теплоты. Количество вещества оксида углерода, образовавшегося при этом, равно

1) 3 моль 2) 1,5 моль 3) 4,5 моль 4) 6 моль

Занятие 9

Скорость химической реакции. Химическое равновесие

План изучения темы

1. Скорость гомогенной и гетерогенной реакций.2. Факторы, влияющие на скорость химической реакции.3. Катализаторы, ингибиторы.4. Реакции обратимые и необратимые.5. Факторы, влияющие на смещение химического равновесия.

Тест для самостоятельной работы1. При получении кислорода разложением бертолетовой соли в лаборатории в качестве катализатора используют

1) HgCl2 2) MnO2 3) SiO2 4) Al2O3

2.При комнатной температуре с наибольшей скоростью протекает реакция между

1) NaOH(р-р) и HCl(р-р)

51

Page 50: эл курс подг к егэ Word (2)

2) CuO и H2SO4 (р-р)

3) CaCO3 и HCl(р-р)

4) Zn(тв.) и H2SO4 (р-р)

3. Равновесие химической реакции N2 + O2 ⇄ 2NO – Q будет смещаться в сторону продукта реакции при

1) увеличении концентрации кислорода2) увеличении давления3) уменьшении давления4) понижении температуры

4.При увеличении температуры на каждые 10◦ скорость химической реакции

1) увеличивается на одну и ту же величину2) изменяется периодически3) возрастает примерно в 2 – 4 раза4) возрастает в 10 раз

5. Химическое равновесие наступает тогда, когда

1) концентрации реагирующих веществ станут равными 2) скорость химической реакции станет равной нулю 3) скорости прямой и обратной реакций окажутся равными 4) скорость химической реакции достигнет максимума

6. Вещества, которые ускоряют химические реакции, но при этом не расходуются, называются

1) ингибиторами 2) адсорбентами 3) катализаторами 4) инициаторам

7. С наибольшей скоростью с раствором серной кислоты взаимодействует

1) CaCO3 2) Fe(OH)3 3) KOH 4) Zn8. На скорость химической реакции между раствором серной кислоты и железом не оказывает влияния

1) концентрация кислоты 2) измельчение железа 3) температура реакции

52

Page 51: эл курс подг к егэ Word (2)

4) увеличение давления

9. Смещению химического равновесия вправо в системе SO2(г) + Cl2(г) ⇄ SO2Cl2(г) + Q способствует

1) увеличение температуры2) уменьшение давления3) увеличение концентрации хлора4) уменьшение концентрации оксида серы (IV)

10. Изменение площади поверхности соприкосновения реагирующих веществ в большей мере влияет на скорость реакции между

1) CO и O 2) CaCO3 и HCl 3) NaCl(р-р) и AgNO3(р-р)

4) H2 и Cl2

11. Введение катализатора в систему, находящуюся в равновесие, 1) приводит к смещению равновесия в сторону продуктов реакции 2) приводит к смещению равновесия в сторону исходных веществ 3) не влияет на равновесие 4) приводит к появлению побочных продуктов реакции

12. Химическое равновесие в реакции CO2 (г) + C(тв) ⇄ 2CO(г) – Q сместится вправо при

1) повышении давления2) понижении температуры3) повышении концентрации CO4) повышении температуры

13. Эндотермической является реакция 1) N2 + H2 = 2NH3

2) N2 + O2 = 2NO 3) 4NH3 + 3O2 = 2N2 + 6H2O 4) NH3 + HCl = NH4Cl14. С наибольшей скоростью с раствором гидроксида натрия взаимодействует

1) Al 2) Cl2 3) Al(OH)3 4) CuSO4

15. Эндотермической является реакция

1) MgCO3 = MgO + CO2

53

Page 52: эл курс подг к егэ Word (2)

2) 2H2 + CO = CH3OH 3) O2 + 2CO = 2CO2

4) CH4 + 2O2= CO2 + 2H2O

16. Эндотермической является реакция

1) Ca + 2H2O = 2Ca(OH)2 + H2

2) Ca (HCO3)2 = CaCO3 + CO2 + H2O 3) CaC2 + 2H2O = 2Ca(OH)2 + C2H2

4) CaCO3 + 2HCl = Ca Cl2 + H2O + CO2

17. При одновременном повышении температуры и понижении давлении химическое равновесие смещается влево в системе

1) CO2 + C(тв) ⇄ 2CO – Q 2) H2 + Cl2 ⇄ 2HCl + Q 3) 2CO + O2 ⇄ 2 CO2+ Q 4) N2 + O2 ⇄ 2NO – Q

18. При комнатной температуре с наибольшей скоростью происходит реакция между

1) HCl (р-р) и MnO2

2) Zn и HCl (р-р) 3) CaCO3 и HCl (р-р) 4) HCl (р-р) и KOH (р-р)

19. При обычных условиях с наибольшей скоростью происходит

взаимодействие между

1) C2H5OH и Na 2) Fe и S 3) N2 и O2

4) Na2CO3 и BaCl2

20. Скорость химической реакции 2NO2 → 2NO + O2 – 58 кДж не зависит от изменения

1) концентрации диоксида азота2) давления в системе3) концентрации кислорода4) температуры

21. К экзотермическим реакциям относится реакция

54

Page 53: эл курс подг к егэ Word (2)

1) C2H4 → C2H2 + H2

2) 2NaNO3 → 2 NaNO2 + O2

3) C3H8 → C3H6 + H2

4) 2H2O + O2 + 4 NO2→ 4HNO3

22. Для увеличения скорости химической реакции Mg(тв) + 2H+

→ Mg2+ + H2(г)

необходимо

1) добавить несколько кусочков магния2) увеличить концентрацию ионов водорода3) уменьшить температуру4) увеличить концентрацию ионов магния

23. К экзотермическим реакциям относится реакция

1) CH4+ 4Cl2 → CCl4+ 4HCl 2) C + H2O → CO + H2

3) C20H42 → C10H22 + C10H10

4) Li2CO3 → Li2O + CO2

24. Равновесие химической реакции N2 + O2 ⇄ 2NO – Q будет смещаться в сторону продукта реакции при

1) увеличении концентрации кислорода2) увеличении давления3) уменьшении давления4) понижении температуры

25. С наибольшей скоростью с водородом реагирует 1) Cl2 2) F2 3) S 4) C

26. Смещению химического равновесия в сторону исходных веществ в реакции 2SO2 + O2 ⇄ 2SO3 + Q будет способствовать

1) уменьшение давления2) уменьшение температуры3) увеличение концентрации SO2

4) уменьшение концентрации SO3

27. С наибольшей скоростью с водородом может реагировать

1) O2 2) Li 3) Si 4) Se

55

Page 54: эл курс подг к егэ Word (2)

28. Экзотермической является реакция

1) C2H4 → C2H2 + H2

2) C4H10→ C2H4 + C2H6 3) 2AgNO3 → 2Ag + 2NO2 + O2

4) CH4 + 2O2 → CO2 + 2H2O

29. Экзотермической является реакция

1) FeCO3 → FeO + CO2

2) C2H6→ C2H4 + H2

3) C2H6 + Cl2 → C2H5Cl + HCl 4) 2CH4 → C2H4 + 3H2

30. Равновесие химической реакции C2H5OH + CH3COOH ⇄ CH3COOC2H5 + H2O – Q смещается в сторону продуктов реакции при

1) добавлении воды 2) уменьшении концентрации уксусной кислоты 3) увеличении концентрации эфира 4) увеличении температуры

Занятие 10 Растворы. Теория электролитической диссоциации

План изучения темы

1. Понятие о растворах. Растворимость веществ в воде. 2. Количественная характеристика растворов. 3.Электролитическая диссоциация. Степень диссоциации. 4. Диссоциация кислот, оснований, солей в водных растворах. 5. Реакции обмена в водных растворах.

Тест для самостоятельной работы

1. Одновременно могут находиться в растворе

1) катион натрия и гидроксид- -ион2) катион цинка и гидроксид- ион3) катион алюминия и гидроксид - ион4) катион меди и гидроксид - ион

2. При диссоциации 1 моль фосфата натрия число моль ионов равно

56

Page 55: эл курс подг к егэ Word (2)

1) 1 2) 2 3) 3 4) 4

3. Вещество, которое не является электролитом, имеет формулу

1) NaOH 2) NaCl 3) (C2H5)2O 4) CH3COOH

4. Степень диссоциации вещества в растворе составляет 95%. Вещество является

1) сильным электролитом2) электролитом средней силы3) слабым электролитом4) неэлектролитом

5. Наибольшее число ионов образует при диссоциации 1 моль

1) FeCl3 2) NaHCO3 3) Al2(SO4)3 4) (FeOH)Cl

6. В качестве катионов только ионы Н+ образуются при диссоциации

1) CH3 – COH 2) HNO3 3) NH4Cl 4) KHSO4

7. В качестве анионов только ионы ОН– образуются при диссоциации

1) CH3OH 2) Zn(OH)Br 3) NaOH 4) CH3COOH8. В качестве катионов только ионы Н+ образуются при диссоциации 1) CHCl3 2) HCl 3) NH4Cl 4) NaHCO3

9. В качестве анионов только ионы ОН– образуются при диссоциации

1) Ca(OH)Cl 2) KOH 3) C2H5OH 4) HCOH

10. Число моль ионов, образующихся при диссоциации 1 моль сульфита натрия, равно

1) 1 2) 2 3) 3 4) 4

11. Только анионы ОН– образуются при диссоциации

1) Ba(OH)2 2) CH3 – OH 3) (CuOH)2CO3 4) HCOOH

12. Электролитом не является

57

Page 56: эл курс подг к егэ Word (2)

1) расплав гидроксида натрия 2) азотная кислота 3) раствор гидроксида натрия 4) этиловый спирт

13. Сильными элктролитами являются

1) HCOOH и Cu(OH)2

2) Ca3(PO4)2 и Mg(OH)2

3) K2CO3 и Pb(OH)2

4) KHCO3 и H2SO4

14. К электролитам относится каждое из двух веществ

1) Cu(ОН)2 и AgCl 2) NaCl и FeS 3) CaCl2 и BaSO4

4) K2SO4 и NaNO3

15. В качестве катионов только ионы Н+ образуются при диссоциации

1) NaHCO3 2) C2H5OH 3) NH3 4) H2SO4

16. Среди веществ: угарный газ, соляная кислота, гидроксид натрия, глицерин, свинец – число электролитов и неэлектролитов равно 1) 2 и 3 2) 3 и 2 3) 1 и 4 4) 4 и 1 17. Одинаковое количество катионов и анионов образуется при диссоциации

1) CH3COONa 2) K2SO4 3) AlCl3 4) Ba(NO3)2

18. Одновременно существовать в водном растворе не могут ионы

1) Zn2+ и OH–

2) Zn2+ и Cl–

3) Cu2+ и NO3–

4) Fe2+ и SO42–

19. Наиболее сильным электролитом является 1) H2S 2) HF 3) NaF 4) H2SiO3

20. Диссоциация по трем ступеням возможна в растворе

1) хлорида алюминия 2) нитрата алюминия 3) ортофосфата калия 4) ортофосфорной кислоты

58

Page 57: эл курс подг к егэ Word (2)

21. К 60 г 20% раствора сульфата натрия добавили 60 мл воды. Массовая доля сульфата натрия в полученном растворе равна _________%. (Запишите число с точностью до целых.)

22. При взаимодействии 2 моль цинка и 200 г 19,6%-ного раствора серной кислоты образуется соль массой  _________ г. (Запишите число с точностью до целых.)

23. Какую массу оксида кальция необходимо взять для приготовления 200 г раствора гидроксида кальция с массовой долей 7,4 % ? Ответ _________ г. (Запишите ответ с точностью до десятых.) 24. Масса соли, которая содержится в изотоническом растворе массой 500 г, содержащим 0,9% по массе поваренной соли, равна _________ г. (Запишите ответ с точностью до десятых.)

25. Объём воды, добавленный к 70 г 10%-ного раствора поваренной соли для получения 5%-ного её раствора, равен _________ мл. (Запишите ответ с точностью до целых.)

26. При взаимодействии 2 моль железа и 200 г 9,8 %-ного раствора серной кислоты образуется соль массой  _________ г. (Запишите число с точностью до целых.)27. Смешали 100 мл 14,5%-ного раствора сульфата меди (II) (r=1,1 г/мл) и 40 г 25%-ного раствора гидроксида натрия. Масса образовавшегося осадка равна _________ г. (Запишите ответ с точностью до целых.)

28. Смешали 200 г 15%-ного раствора и 300 г 20%-ного раствора глюкозы. Массовая доля глюкозы в полученном растворе равна __________%. (Запишите ответ с точностью до целых.)

29. Рассчитайте массу осадка, полученного в результате взаимодействия 100г 16%-ного раствора сульфата меди(II) с избытком раствора гидроксида калия. Ответ: __________ г. (Запишите число с точностью до целых.)

30. К 250г 8%-ного раствора гидроксида натрия добавили 150 мл воды. Рассчитайте массовую долю (в %) гидроксида натрия в полученном растворе. Ответ: __________ %. (Запишите число с точностью до целых.)

Занятие 11Важнейшие классы неорганических соединений

59

Page 58: эл курс подг к егэ Word (2)

План изучения темы

6. Классификация неорганических соединений.7. Номенклатура, классификация, получение, физические и

химические свойства оксидов.8. Номенклатура, классификация, получение, физические и

химические свойства оснований.9. Номенклатура, классификация, получение, физические и

химические свойства кислот.10.Номенклатура, классификация, получение, физические и

химические свойства солей.11.Получение, физические и химические свойства амфотерных

оксидов и амфотерных гидроксидов.12.Генетическая взаимосвязь неорганических соединений.

Тест для самостоятельной работы

1. Оксид азота (II) является оксидом

1) основным2) кислотным3) амфотерным4) несолеобразующим

2. К кислым солям относятся:

1) Сa(NO3)2 2) Mg(OH)Cl 3) KHCO3 4) NH4Cl

3. Формулы только основных оксидов расположены в ряду:

1) СrO, CaO, Mn2O7

2) CO2, SO2, CuO 3) SrO, MnO, CrO 4) SiO2, Na2O, FeO

4. К амфотерным оксидам относится: 1) CrO3 2) CrO 3) CO 4) Cr2O3

5. Основные свойства наиболее выражены у оксида:

1) магния2) кальция

60

Page 59: эл курс подг к егэ Word (2)

3) стронция4) бария

6. К основным солям относится:

1) NH4HSO4

2) Fe(OH)SO4

3) K2SO4

4) KHCO3

7. Формулы только гидроксидов неметаллов расположены в ряду:

1) Mg(OH)2, Mn(OH)2, NaOH2) H2SO4, HNO3, HCl3) H2SO3, H2SO4, H2S4) HClO4, H3PO4, H2SO4

8. Кислотный оксид может образовать элемент:

1) барий 2) марганец 3) калий 4) медь

9. Кислотные свойства оксидов в ряду: N2О5 – Р2О5 – As2O5 – Sb2O5,

1) убывают2) возрастают3) не изменяются4) отсутствуют

10. Сила кислот в ряду HNO3 – HPO3 – HAsO3

1) не изменяются2) вначале убывает, затем возрастает3) возрастает4) убывает

11. Сила кислот увеличивается в ряду

1) HCl, H2S, HClО4 2) HI, HBr, HCl 3) H2CO3, HF, HBr

61

Page 60: эл курс подг к егэ Word (2)

4) H2SO3, H3PO4, H2SO4

12.Оксид меди (II) реагирует с каждым из двух веществ: 1) серная кислота и вода2) водород и гидроксид бария3) водород и соляная кислота4) сульфат натрия и хлор

13. Оксид серы (IV) реагирует с каждым из двух веществ:

1) кислород и серная кислота2) сероводород и кислород3) вода и сульфат калия4) гидроксид натрия и кальций

14. Алюминий взаимодействует с каждым из двух веществ:

1) Cl2 и NH3 2) HCl и NaOH 3) H2SO4 и Mg(OH)2 4) NaCl и HBr

15. Соляная кислота реагирует с каждым из двух веществ:

1) карбонат натрия и фосфат натрия 2) сульфат натрия и гидроксид натрия3) железо и перманганат калия4) оксид серы (IV) и оксид бария

16. Сульфат меди (II) можно получить при взаимодействии:

1) CuCl2 и CaSO4 2) CuO и H2SO4 3) Cu(OH)2 и Na2SO4 4) CuCl2 и K2SO4

17. С раствором гидроксида натрия при обычных условиях не реагирует:

1) оксид углерода (II)2) оксид углерода (IV)3) оксид фосфора (V)4) оксид азота (IV)

62

Page 61: эл курс подг к егэ Word (2)

18. В водном растворе с углекислым газом реагирует:

1) AgNO3

2) BaSO4

3) CaCO3

4) Ba(NO3)2

19. Гидроксид бериллия проявляет амфотерные свойства при взаимодействии с

1) HCOOH и H2S2) KOH и Ca(OH)2

3) H2SO4 и SO3

4) HNO3 и NaOH

20. Сульфат аммония реагирует с

1) Cu(NO3)2

2) NO2

3) HCl4) Ca(OH)2

21. Установите соответствие между формулой вещества и классом (группой) неорганических соединений

ФОРМУЛА ВЕЩЕСТВА

КЛАСС (ГРУППА) НЕОРГАНИЧЕСКИХ

СОЕДИНЕНИЙА) Ba(OH)2 1) кислотный оксидБ) HClO4 2) бескислородная кислотаВ) NH4H2PO4 3) кислая сольГ) Mn2O7 4) оснóвный оксид

5) кислородсодержащая кислота6) основание

22. Установите соответствие между формулой вещества и классом (группой) неорганических соединений

ФОРМУЛА ВЕЩЕСТВА КЛАСС (ГРУППА) НЕОРГАНИЧЕСКИХ

СОЕДИНЕНИЙА) Сa(OH)2 1) амфотерный гидроксид

63

Page 62: эл курс подг к егэ Word (2)

Б) HCl 2) бескислородная кислотаВ) NaHCO3 3) кислая сольГ) Be(OH)2 4) оснóвный оксид

5) кислородсодержащая кислота6) основание

23. Установите соответствие между формулой вещества и классом (группой) неорганических соединений

ФОРМУЛА ВЕЩЕСТВА КЛАСС (ГРУППА) НЕОРГАНИЧЕСКИХ

СОЕДИНЕНИЙА) CrO3 1) кислородсодержащая кислотаБ) HClO4 2) основный оксидВ) Mg(OH)Cl 3) кислотный оксидГ) NH4NO3 4) основная соль

5) кислая соль6) средняя соль

24. Установите соответствие между формулами веществ , указанными попарно и классами (группами) неорганических соединений, к которым они принадлежат.

ФОРМУЛА ВЕЩЕСТВ КЛАСС (ГРУППА) НЕОРГАНИЧЕСКИХ

СОЕДИНЕНИЙА) Zn(OH)2 и K2SO4 1) средняя соль, кислая сольБ) Na2SO4 и KHCO3 2) кислота, амфотерный оксидВ) HClO и CrO3 3) амфотерный гидроксид, сольГ) H3PO4 и BeO 4) кислота, кислотный оксид

5) основание, кислотный оксид6) основание, средняя соль

25. Установите соответствие между оксидом элемента и общей формулой его гидроксида.

ФОРМУЛА ВЕЩЕСТВА ФОРМУЛА ГИДРОКСИДАА) Cl2O7 1) НЭО4

Б) Rb2O 2) ЭОНВ) CaO 3) НЭО3

64

Page 63: эл курс подг к егэ Word (2)

Г) SO2 4) Н2ЭО3

5) Э(ОН)2

6) Н2ЭО4

26. Установите соответствие между исходными веществами и продуктами реакции между ними.

ИСХОДНЫЕ ВЕЩЕСТВА ПРОДУКТЫ РЕАКЦИИ

А) KOH + SO3(изб) → 1) KHSO4

Б) KOH + SO3 (недост) → 2) K2CO3 и H2OВ) KOH + CO2(изб) → 3) K2SO3 и H2OГ) KOH + CO2 (недост) → 4) K2SO4 и H2O

5) KHCO3

6) KHSO3

27. Установить соответствие между исходными веществами и образующимся в результате реакции газом.

ИСХОДНЫЕ ВЕЩЕСТВА ВЫДЕЛЯЮЩИЙСЯ ГАЗ

А) NaCl(тв) + H2SO4(конц) 1) Cl2

Б) NH4Cl + Ba(OH)2 → 2) HClВ) NH4NO3 3) NH3

Г) (NH4)2CO3 + HCl → 4) N2O5) CO2

6) SO2

28. С гидроксидом бария реагируют:

1) гидроксид цинка 2) гидроксид натрия3) сульфат натрия4) нитрат калия5) оксид кальция 6) оксид серы (IV)

65

Page 64: эл курс подг к егэ Word (2)

29. С соляной кислотой реагируют:

1) гидроксид меди (II) 2) гидроксид фосфора (V)3) сульфат натрия4) сульфид калия5) оксид кальция 6) оксид серы (VI)

30. С раствором сульфата калия реагируют:

1) гидроксид меди (II)2) гидроксид натрия3) гидроксид бария4) азотная кислота5) оксид кальция 6) хлорид стронция

Занятие 12 Гидролиз неорганических и органических соединений

План изучения темы1) Гидролиз солей.2) Усиление и ослабление гидролиза солей.3) Гидролиз органических соединений.4) Гидролиз простых и сложных неорганических соединений.

Тесты для самостоятельной работы

1. Среда раствора в результате гидролиза хлорида аммония:

1) слабощелочная 2) кислая 3) сильнощелочная 4) нейтральная

2. Гидролизу не подвергается

66

Page 65: эл курс подг к егэ Word (2)

1) крахмал 2) карбид алюминия 3) сульфат цинка 4) сульфат натрия

3. Кислая соль образуется в водном растворе

1) сульфата калия 2) бромида алюминия 3) карбоната калия 4) нитрата цинка

4. Бесцветный раствор соли Ca(NO3)2 окрасится лакмусом в цвет: 1) красный 2) зеленый 3) синий 4) желтый

5. Добавление какого вещества к воде приведет к уменьшению его рН ниже 7

1) оксид бария 2) хлорид кальция 3) хлорид алюминия 4) сульфат калия6. Гидролиз солей усиливается:

1) при повышении давления 2) при понижении давления 3) при разбавлении раствора 4) при повышении концентрации соли

7. При гидролизе клетчатки (крахмала) могут образовываться:

1) глюкоза 2) только сахароза 3) только фруктоза 4) углекислый газ и вода

8. Среда раствора в результате гидролиза иодида меди (II):

1) нейтральная; 2) слабощелочная; 3) кислая 4) щелочная

67

Page 66: эл курс подг к егэ Word (2)

9. Гидролизу не подвергается 1) нитрат натрия 2) нитрат цинка 3) сахароза 4) триглицерид

10. Основная соль образуется при гидролизе 1) сульфата аммония 2) сульфата алюминия 3) нитрата аммония 4) фторида серебра

11. Фенолфталеин становится малиновым в водном растворе 1) сульфата железа (III) 2) сульфата натрия 3) сульфида натрия 4) сульфата цинка

12. Гидролиз солей усиливается 1) при добавлении в раствор этой же соли

2) при понижении температуры 3) при повышении температуры 4) при повышении давления

13. При гидролизе жиров образуются:

1) одноатомные спирты и карбоновые кислоты; 2) альдегиды и карбоновые кислоты; 3) глицерин и минеральные кислоты; 4) глицерин и карбоновые кислоты.

14. Среда раствора в результате гидролиза сульфата аммония:

1) слабощелочная 2) щелочная

3) нейтральная 4) кислая

15. Какая соль подвергается гидролизу только по катиону:

1) хлорид натрия; 2) ацетат аммония;

3) карбонат натрия; 4) нитрат алюминия.

68

Page 67: эл курс подг к егэ Word (2)

16. Чтобы ослабить или прекратить гидролиз раствора хлорида железа (III), необходимо добавить к его раствору

1) гидроксид натрия 2) дистиллированной воды 3) соляной кислоты 4) твердой соли NaCl

17. При гидролизе сахарозы образуется:

1) α- и β-глюкоза 2) мальтоза

3) глюкоза и рибоза 4) глюкоза и фруктоза

18. Среда раствора силиката калия:

1) нейтральная 2) щелочная

3) слабокислая 4) кислая

19. Гидролизу и по катиону, и по аниону подвергается

1) хлорид алюминия 2) сульфит натрия

3) гидросульфид аммония 4) нитрат аммония

20. Бесцветный раствор какой соли окрасится метилоранжем в розовый цвет:

1) нитрат цинка 2) карбонат натрия

3) нитрат бария 4) нитрат кальция

21. Гидролиз солей усиливается:

1) при понижении температуры2) при уменьшении концентрации соли3) при понижении давления

69

Page 68: эл курс подг к егэ Word (2)

4) при повышении давления

22. Осадок, полученный при обработке хлорида кремния (IV) водой, прокалили. Укажите молярную массу твердого вещества после прокаливания.

23. Газ, полученный при обработке фосфида магния водой, сожгли в избытке кислорода. Укажите молекулярную массу твердого вещества, полученного при этом.

24. Газ, полученный при обработке нитрида кальция водой, сожгли. Укажите молярную массу газа, образующегося при этом.

25. Газ, полученный при обработке нитрида магния водой, пропустили над раскаленным порошком оксида меди (II). Укажите молярную массу твердого вещества, полученного при этом.

26. Газ, полученный при обработке гидрида натрия водой, пропустили при нагревании над оксидом меди (II). Укажите молярную массу твердого вещества, полученного при этом.

27. Слили растворы нитрата железа (III) и карбоната натрия. Составьте

уравнение химической реакции и опишите внешние признаки 28. Слили растворы хлорида хрома (III) и сульфида натрия. Составьте

уравнение химической реакции и опишите внешние признаки.29. Слили растворы хлорида железа (III) и карбоната калия. Составьте уравнение химической реакции и опишите внешние признаки.

30. Слили растворы нитрата алюминия и карбоната натрия. Составьте уравнение химической реакции и опишите внешние признаки.

Занятие 13Металлы

План изучения темы 1. Положение металлов в периодической системе химических

элементов Д. И. Менделеева.2. Физические свойства металлов.3. Общие химические свойства металлов.4. Химический характер оксидов и гидроксидов металлов.5. Общие способы получения металлов.6. Электролиз расплавов и водных растворов соединений металлов.7. Коррозия металлов. Защита металлов от коррозии.8. Металлы IА- IIА групп.

70

Page 69: эл курс подг к егэ Word (2)

9. Алюминий и его соединения.10. Железо и хром – металлы побочных подгрупп.

Тесты для самостоятельной работы

А1. Какая электронная конфигурация соответствует атому калия:

1) 1s22s22p63s23p63d1 2) 1s22s22p63s23p64s1 3) 1s22s22p63s23p64s2

4) 1s22s22p63s23p64s24p1

А2. Про электроны, находящиеся в кристаллической решетке калия, можно сказать, что они:

1) свободно перемещаются между всеми ионам и атомами2) попарно закреплены между всеми ионами и атомами3) жестко закреплены между всеми ионам и атомами4) перемещаются от иона к иону

А3. Натрий нельзя получить электролизом:

1) расплава NaBr2) расплава NaOH3) расплава NaCl4) раствора NaOH

А4. Раствор гидроксида натрия взаимодействует с каждым веществом, указанным в ряду:

1) алюминий, оксид алюминия, гидроксид алюминия, оксид кальция2) цинк, оксид цинка, гидроксид цинка, оксид бария3) цинк, гидроксид цинка, хлорид цинка, соляная кислота4) железо, гидроксид железа (III), оксид железа (II), хлорид железа (II)

А5. Реакция между растворами карбоната калия и серной кислотой идет до конца, так как:

1) в результате реакции образуется сульфат калия2) ее используют для обнаружения в растворе карбонат-ионов3) эта реакция ионного обмена4) в результате образуется оксид углерода (IV)

А6. Элементу с зарядом ядра атома +20 соответствует высший оксид:

71

Page 70: эл курс подг к егэ Word (2)

1) Э2О 2) ЭО 3) Э2О3 4) ЭО2

А7. Гидроксид кальция может реагировать с каждым из веществ,

указанных в паре:

1) азотная кислота, оксид меди (II)2) соляная кислота, оксид серы (IV)3) соляная кислота, нитрат натрия4) азотная кислота, оксид азота (I)

А8. Наиболее сильным восстановителем среди металлов II A группы

является:

1) барий 2) кальций 3) стронций 4) магний

А9. Осадок выпадает при пропускании углекислого газа через раствор:

1) хлорида кальция

2) нитрата кальция

3) гидроксида кальция

4) гидрокарбоната кальция

А10. Атом элемента имеет на 6 электронов больше, чем ион Al3+ . химический знак этого элемента:

1) S 2) Si 3) P 4) ClА11. Водород не выделяется при взаимодействии

1) Al + NaOH (р-р) → 2) Al + HNO3 (р-р) →

3) Al + H2SO4 (р-р) →4) Al + HCl (р-р) →

12. В схемеAl(OH)3 Al2O3 Al NaAlO2 AlCl3

требуется провести электролиз расплава на этапе:

1) 1 2) 2 3) 3 4) 4

13. Электронная конфигурация иона Fе2+

1) 1s22s22p63s23p63d6

2) 1s22s22p63s23p63d5

3) 1s22s22p63s23p63d64s2 4) 1s22s22p63s23p63d4

72

Page 71: эл курс подг к егэ Word (2)

14. В схеме превращений FeCl3 Fe (OH)3 FeCl3 AgCl

веществами А, Б, В являются соответственно

1) H2O, NaOH, AgNO3 2) KOH, HCl, AgNO3 3) H2O, HCl, AgNO3

4) NaOH, KCl, AgNO3

15. Осадок не выпадает при добавлении к раствору хлорида хрома (III) раствора

1) гидроксида натрия (недостаток) 2) фторида натрия 3) фосфата натрия 4) серной кислоты

16. Укажите степень окисления восстановителя в химической реакции, схема которой

Fe2(SO4)3 Fe2O3 + SO2 + O2

1) +6 2) +3 3) –2 4) +217. Для качественного обнаружения катионов Fe2+ используют реакцию с

1) гексацианоферратом (II) калия2) желтой кровяной солью3) роданидом аммония4) гексацианоферратом (III) калия

18. Чугун в промышленности получают1) электролизом раствора нитрата железа (II)2) восстановлением кальцием из оксида железа (II)3) выплавкой в доменной печи4) прокаливанием минерала сидерита (FeCO3)

19. В железной цистерне нельзя хранить реактив1) H2SO4 (конц.) 2) HNO3 (конц.) 3) HCl (конц.) 4) олеум

20. Масса соли, полученной в результате взаимодействия 150 г 10,95% - ного раствора соляной кислоты и 13 г хрома, равна

1) 30,75 г 2) 55,35 г 3) 15,375 г 4) 27,675 г

21. Установите соответствие между названием вещества и

73

Page 72: эл курс подг к егэ Word (2)

электролитическим способом его получения.

НАЗВАНИЕ ВЕЩЕСТВА ЭЛЕКТРОЛИЗ А) кислород 1) водного раствора CuSO4

Б) щелочь 2) водного раствора CaCl2

В) водород 3) водного раствора CuBr2

Г) бром 4) расплава NaI5) водного раствора CuСl2

6) расплава SiO2

22. Установите соответствие между формулой электролита и продуктом, образующимся на инертном аноде при электролизе его водного раствора.

ФОРМУЛА ЭЛЕКТРОЛИТА

ПРОДУКТ ЭЛЕКТРОЛИЗА

А) CsCl 1) хлорБ) HgCl2 2) кислородВ) Rb3PO4 3) водородГ) HgSO4 4) серная кислота

5) фосфорная кислота6) соляная кислота

23. Установите соответствие между формулой вещества и продуктом, который образуется на инертном аноде в результате электролиза водного раствора этого вещества.

ФОРМУЛА ВЕЩЕСТВА ПРОДУКТ ЭЛЕКТРОЛИЗА А) BaBr2 1) NO2

Б) K2SO4 2) Br2

В) Ca(NO3)2 3) Cl2

Г) CuCl2 4) O2

5) SO2

6) H2

24. Установите соответствие между формулой вещества и продуктом, который образуется на катоде в результате электролиза его водного раствора.

ФОРМУЛА ВЕЩЕСТВА ПРОДУКТ ЭЛЕКТРОЛИЗА А) Cu(NO3)2 1) водородБ) CaCl2 2) кислородВ) K2SO4 3) металлГ) CuSO4 4) хлор

5) оксид серы (IV)

74

Page 73: эл курс подг к егэ Word (2)

6) оксид азота (IV)

25. Установите соответствие между формулой вещества и продуктом, который образуется на инертном аноде в результате электролиза его водного раствора.

ФОРМУЛА ВЕЩЕСТВА ПРОДУКТ ЭЛЕКТРОЛИЗА А) LiBr 1) O2

Б) Na2SO4 2) Cl2

В) CuCl2 3) Br2

Г) Cu(NO3)2 4) NO2

5) SO2

26. Установите соответствие между формулой вещества и простым веществом, которое образуется на катоде в результате электролиза его водного раствора.

ФОРМУЛА ВЕЩЕСТВА ПРОДУКТ ЭЛЕКТРОЛИЗА А) Li2S 1) MgБ) Hg(NO3)2 2) H2

В) CuSO4 3) HgГ) MgCl2 4) Li

5) S6) Cu

27. Установите соответствие между формулой вещества и продуктом, который образуется на инертном аноде в результате электролиза его водного раствора.

ФОРМУЛА ВЕЩЕСТВА ПРОДУКТ ЭЛЕКТРОЛИЗА А) K2S 1) Cl2

Б) NiCl2 2) O2

В) NiF2 3) H2

Г) NiSO4 4) S

5) SO2

6) HF

28. Установите соответствие между формулой вещества и продуктом, который образуется на катоде в результате электролиза его водного раствора.

ФОРМУЛА ВЕЩЕСТВА ПРОДУКТ ЭЛЕКТРОЛИЗА А) CrCl3 1) водородБ) AgNO3 2) металлВ) Na3PO4 3) металл и водородГ) KCl 4) кислород

5) хлор

75

Page 74: эл курс подг к егэ Word (2)

6) азот

29. Какой объем газа (н.у.) выделится при действии на 20,84 г известняка (содержащего 4% не реагирующих с кислотой примесей) 73 г 24% раствора соляной кислоты?

30. 4,8 г металлического магния обработали 200 г 19,6% раствора серной кислоты. Каков объем газа (н.у.) выделился при этом.

Занятие 14Неметаллы

План изучения темы1) Положение неметаллов в периодической системе химических

элементов Д. И. Менделеева.2) Строение простых веществ- неметаллов.3) Физические и химические свойства неметаллов.4) Водородные соединения неметаллов.5) Характеристика оксидов и гидроксидов неметаллов.6) Галогены. Хлор, его получение, свойства.7) Кислород, его получение, свойства.8) Сера и ее важнейшие соединения.9) Азот. Соединения азота: аммиак, оксиды азота.10) Соли аммония, получение и свойства.11) Азотная кислота, ее получение, свойства.12) Фосфор и его важнейшие соединения, оксиды, гидроксиды,

соли фосфорной кислоты.13.Углерод и его важнейшие неорганические соединения: оксиды,

гидроксиды, соли.14.Кремний и его важнейшие соединения: оксиды. Гидроксиды,

соли.

Тесты для самостоятельной работы

А1. Электронная формула внешнего энергетического уровня атома элемента, имеющего формулы газообразного водородного соединения RH и высшего оксида R2O7 (n – номер внешнего энергетического уровня)

1) ns2np2 2) ns2np3 3) ns2np4 4) ns2np5

А2. Электронную формулу 1s22s22p63s23p3 имеет атом

1) кремния 2) хлора 3) фосфора 4) фтора

76

Page 75: эл курс подг к егэ Word (2)

А3. Какой из частиц соответствует электронная формула 1s22s22p6

1) Cl– 2) C4+ 3) Si4+ 4) C2+

А4. Молекулярное строение имеет вещество, формула которого

1) I2 2) Si 3) B 4) SiO2

А5. Неметаллические свойства у элементов А групп усиливаются

1) слева направо и в группах сверху вниз2) справа налево и в группах сверху вниз3) справа налево и в группах снизу вверх4) слева направо и в группах снизу вверх

А6. Все галогены проявляют в соединениях степень окисления

1) +7 2) +5 3) +1 4) –1

А7. Двухосновной бескислородной кислотой является

1) бромоводородная 2) сернистая 3) сероводородная4) серная

А8. Одновременно не могут находиться в растворе ионы

1) K+, Ca2+, NO3–, Cl–

2) K+, Ca2+, Cl–, F–

3) Na+, Ca2+, NO3–, Cl–

4) Na+, Ba2+, Cl–, OH–

А9. Связь в соединении, образованном атомом водорода и элементом, имеющим распределение электронов в атоме 2,8,6, является

1) ионной 2) ковалентной полярной3) ковалентной неполярной4) металлической

А10. Сила кислот в рядуH2O → H2S → H2Se → H2Te

1) сначала уменьшается, затем возрастает 2) уменьшается3) возрастает

4) не изменяется

77

Page 76: эл курс подг к егэ Word (2)

А11. Сила галогеноводородных кислот возрастает в ряду:

1) HI, HCl, HBr 2) HI, HBr, HCl3) HCl, HBr, HI4) HBr, HI, HCl

А12. Химическая связь в соединении KСlO4

1) ковалентная неполярная2) ковалентная полярная3) ионная4) ковалентная и ионная

А13. В химической реакции Ca + 2H2O → Ca(OH)2 + H2

окислителем

1) Ca0 2) H+1 3) Ca+2 4) O-2

А14. Для очистки водорода от примеси хлороводорода лучше всего использовать

1) H2SO4 (конц.) 2) CuO 3) H2O (ж) 4) КОН (р-р) А15. Для получения газообразного хлороводорода из хлорида калия с наибольшим выходом следует использовать

1) H2SO4 (конц., хол.) 2) H2SO4 (разб., хол.) 3) H2SO4 (конц., гор.)4) H2SO4 (разб., гор.)

А16. Степень окисления серы в ряду соединений SO3

2– → HSO–3 → SF4 → SO2

1) увеличивается 2) уменьшается3) не изменяется4) уменьшается, затем возрастает

А17. В молекулярном уравнении реакции Zn + H2SO4 (конц.) → H2S + … сумма коэффициентов равна

78

Page 77: эл курс подг к егэ Word (2)

1) 17 2) 16 3) 18 4) 19

А18 Разбавленная серная кислота – окислитель за счет атомов

1) кислорода 2) водорода 3) серы 4) всех элементов

А19 Отрицательная степень окисления проявляется азотом в

соединении:

1) N2O 2) NO 3) Na3N 4) NO2

А20 В молекуле азота атомы связаны

1) двумя π – и одной σ-связью2) двумя σ – и одной π-связью3) двумя π – и одной водородной связью4) ковалентными связями, образованными по донорно- акцепторному механизму

А21 Фосфор в природе в основном встречается в виде соединения

1) Ca3P2 2) P2O5 3) PCl5 4) Ca3(PO4)2

A22 Молекулярный азот N2

1) электропроводен при низких температурах2) нереакционоспособен при обычных условиях3) легко диссоциирует на атомарный азот уже при комнатной температуре4) легко растворим в воде

А23 Металл в свободном виде образуется при прокаливании

1) NaNO3 2) AgNO3 3) Fe(NO3)3 4) Cu(NO3)2

А24 Скорость прямой реакции N2(г) + 3H2(г) ⇄ 2NH3(г) + Qвозрастает при

1) увеличении концентрации азота2) уменьшении концентрации азота3) увеличении концентрации аммиака4) уменьшении давления

79

Page 78: эл курс подг к егэ Word (2)

А25 Коэффициент перед формулой восстановителя в уравнении

реакции, схема которой

Cu + HNO3 → Cu(NO3)2 + NO + H2O, равен

1) 2 2) 3 3) 4 4) 5

Занятие 15 Углеводороды

План изучения темы1) Классификация углеводородов.2) Предельные углеводороды: алканы, циклоалканы, их

электронное и пространственное строение. Номенклатура, изомерия алканов, их физические и химические свойства. Получение и применение алканов и циклоалканов.

3) Этиленовые углеводороды (алкены), их электронное и пространственное строение. Номенклатура, изомерия, физические и химические свойства. Правило Марковникова.

4) Диеновые углеводороды, особенности их химических свойств.5) Ацетиленовые углеводороды (алкины), их электронное

строение. Номенклатура, изомерия, физические и химические свойства.

6) Ароматические углеводороды (арены). Бензол, электронное и пространственное строение. Гомологи бензола. Физические и химические свойства бензола.

7) Природные источники углеводородов и их переработка.

Тесты для самостоятельной работы

1. В каком ряду все вещества являются гомологами между собой

1) С2Н6, С5Н12, С7Н14

2) С2Н2, С3Н4, С4Н8

3) С2Н4, С3Н4, С4Н8

4) С6Н6, С7Н8, С8Н10

2. В виде цис- и транс- изомеров может существовать

1) пропен 2) 2-метилбутен-1 3) 3-метилбутен-1 4) бутен-2

3. Гомологом пропена является

80

Page 79: эл курс подг к егэ Word (2)

1)C2H4 2) C3H8 3) C5H12 4) C6H4

4. Название вещества, структурная формула которого H CH3

| |H − C − CH = C | | CH3 CH3

1) гексен-2 2) 2-метилпентен-2 3) 3-метилпентен-2 4) 1,4-диметилбутен-25. Число σ-связей в молекуле пропена равно

1) 8 2) 6 3) 4 4) 3

6. Число σ-связей в молекуле тетрабромметана равно

1) 1 2) 2 3) 3 4) 4

7. Название углеводорода, в молекуле которого содержатся 13 d-связей

1) бутан 2) 2-метилбутан 3) 2-метилбутен-1 4) бутин-1

8. Структурными изомерами являются:

1) бутин-1 и бутадиен-1,3 2) 2-метилбутен-1 и пентен-1 3) пентан и 2-метилпентан 4) пентен-1 и циклопентан

9. Изопрен относится к классу:

1) алкенов 2) алкадиенов 3) алкинов 4) алканов

10. В молекуле бензола всего d-связей:

1) 12 2) 14 3) 16 4) 18

11. Межклассовыми изомерами являются:

81

Page 80: эл курс подг к егэ Word (2)

1) 2-метилбутен-1 и циклобутан 2) 3-метилбутен-1 и циклопентан 3) бутен-1 и бутен-2 4) бутин-1 и 2-метилпропан

12. Соединение 2-метилпентен-2 и гексен-1 являются:

1) гомологами 2) структурными изомерами 3) межклассовыми изомерами 4) геометрическими изомерами

13. Число s-связей в молекуле этанола равно

1) 8 2) 6 3) 5 4) 4

14. Число p-связей в молекуле бутадиена-1,3 равно:

1) 1 2) 2 3) 3 4) 4

15. Число третичных атомов в молекуле 2-метилпентана равно

1) 1 2) 2 3) 3 4) 4

16. Число изомеров состава C4H9CI равно

1) 2 2) 3 3) 4 4) 5

17. Число структурных изомеров пентана равно

1) 1 2) 2 3) 3 4) 4

18. Изомерами циклопентана являются

1) циклобутан и циклогексан2) 2-метилбутен-1 и метилциклобутан3) метилциклопропан и циклогексан4) пентин и бутадиен-1,3

19. В молекуле какого вещества все атомы углерода находятся в состоянии sp-гибридизации? 1) этана 2) этена 3) этина 4) этанола

82

Page 81: эл курс подг к егэ Word (2)

20. Гомологами являются

1) бутан и 2-метилбутан2) пентен-2 и пентен-13) бутен-2 и гексен-24) бутан и бутен

21. В молекуле какого вещества все атомы углерода находятся в состоянии sp2-гибридизации?

1) бутен-2 2) гексадиен-1,3 3) стирол 4) толуол

22. Верны ли следующие суждения о свойствах ацетилена?

А. Все атомы углерода в ацетилене находятся в состоянии sp -гибридизации. Б. Ацетилен взаимодействует с аммиачным раствором оксида серебра

1) верно только А 2) верно только Б 3) верны оба суждения 4) оба суждения неверны

23. Для пропана характерны:

1) sp2-гибридизация орбиталей всех атомов углерода в молекуле 2) sp3-гибридизация орбиталей атомов углерода в молекуле 3) окисление раствором перманганата калия 4) реакция нитрования 5) реакция дегидрирования 6) взаимодействие с этанолом

24. Для этина характерны:

1) sp-гибридизация орбиталей атомов углерода в молекуле 2) sp3-гибридизация орбиталей атомов углерода в молекуле 3) окисление раствором перманганата калия 4) хорошая растворимость в воде 5) реакция гидратации 6) реакция с аммиачным раствором оксидом серебра (I)

25. Для толоула характерны:

83

Page 82: эл курс подг к егэ Word (2)

1) sp2-гибридизация орбиталей всех атомов углерода в молекуле 2) sp3-гибридизация орбиталей одного атома углерода в молекуле 3) окисление раствором перманганата калия 4) хорошая растворимость в воде 5) реакция замещения 6) взаимодействие с этанолом

26. Для бензола характерны:

1) sp2 –гибридизация атомов углерода в молекуле 2) единая сопряженная p-система 3) хорошая растворимость в воде 4) взаимодействие с водородом 5) взаимодействие с этанолом 6) взаимодействие с бромоводородом

27. Раствор перманганата калия способен обесцветить вещества 1) бензол 2) метилбензол 3) этан 4) бутен-1 5) дивинил 6) 2-метилбутан

28. Для этилена характерны:

1) sp2-гибридизация атомов углерода в молекуле 2) наличие в молекуле 5d и 1p- связей 3) цис- и транс-изомерия 4) плохая растворимость в воде 5) взаимодействие с раствором KMnO4

6) взаимодействие с уксусной кислотой

29. Для пропена характерны: 1) sp2-гибридизация атомов углерода в молекуле 2) наличие десяти d-связей 3) хорошая растворимость в воде 4) реакция полимеризации 5) реакция поликонденсации 6) взаимодействие с галогеноводородами

29. Метан взаимодействуект с

1) раствором перманганата калия

2) азотной кислотой

84

Page 83: эл курс подг к егэ Word (2)

3) соляной кислотой

4) бромом

5) водой

6) кислородом

30. Стирол взаимодействует с

1) раствором перманганата калия2) бромом3) азотной кислотой4) кислородом5) водородом6) этанолом

Занятие 16 Кислородсодержащие органические вещества

План изучения темы8) Предельные одноатомные спирты. Номенклатура, строение,

физические и химические свойства. Промышленные методы получения метанола и этанола.

9) Многоатомные спирты, их строение и свойства на примере этиленгликоля и глицерина.

10) Фенолы. Номенклатура, строение, физические и химические свойства на примере фенола.

11) Альдегиды. Номенклатура, изомерия, строение, физические и химические свойства, получение.

12) Карбоновые кислоты. Номенклатура, изомерия, строение, физические и химические свойства, получение.

13) Сложные эфиры, их строение, получение, физические и химические свойства. Номенклатура сложных эфиров.

14) Углеводы, классификация, строение, физические и химические свойства, применение.

Тесты для самостоятельной работы

1. Карбонильную группу содержит молекула

1) этанола 2) диметилкетона 3) этандиола 4) анилина

85

Page 84: эл курс подг к егэ Word (2)

2. Гидролизу не подвергается 1) сахароза 2) глюкоза 3) крахмал 4) целлюлоза 3. Для фенола нехарактерно взаимодействие с

1) натрием 2) гидроксидом натрия 3) гидроксидом меди (II) 4) хлоридом железа (III)

4. Кислотные свойства наиболее выражены у кислоты 1) уксусной 2) хлоруксусной 3) муравьиной 4) фторуксусной

5. Кислотные свойства наиболее выражены у

1) этанола 2) уксусной кислоты 3) муравьиной кислоты 4) фенола

6. При гидролизе метилформиата образуется:

1) метаналь и метанол 2) муравьиная кислота и метанол 3) метаналь и муравьиная кислота 4) уксусная кислота и метанол

7. Вещество состава C3H6O2 может быть отнесено к классу

1) предельных двухатомных спиртов 2) углеводов 3) простых эфиров 4) предельных карбоновых кислот и сложных эфиров

8. При щелочном гидролизе этилацетата образуются 1) CH3COOH и CH3ONa 2) CH3COOH и C2H5ONa 3) CH3COONa и C2H5OH 4) CH3COONa и C2H5ONa 9. Для глицерина характерно взаимодействие с

86

Page 85: эл курс подг к егэ Word (2)

1) Cu 2) Cu(OH)2 3) FeCl3 4) NaOH

10. Фенол взаимодействуют с каждым веществом пары

1) FeCl3, NaCl 2) Na, HCl 3) NaOH, Br2

4) HNO3, HCl

11. При взаимодействии уксусной кислоты с аммиаком при обычных условиях образуется:

1) амид уксусной кислоты 2) ацетат аммония 3) сложный эфир 4) альдегид12. К спиртам относится вещество, формула которого

1) C6H5OH 2) CH2OH – CH2OH 3) CH3 – CH(CH3) – COOH 4) CH3 – CH(CH3) – CHO

13. Глицерин не взаимодействует с

1) HNO3 2) Cu(OH)2 3) Na2CO3 4) Na

14. Взаимодействуют между собой

1) уксусная кислота и сульфат натрия 2) фенол и карбонат натрия 3) глицерин и уксусная кислота 4) формальдегид и оксид меди (II)

15. Для предельных одноатомных спиртов не характерно взаимодействие с

1) Na 2) NaOH 3) HCl 4) CH3COOH

16. С аммиачным раствором оксида серебра взаимодействует каждое вещество ряда:

1) этанол и этаналь 2) глюкоза и уксусная кислота 3) глюкоза и муравьиная кислота

87

Page 86: эл курс подг к егэ Word (2)

4) формальдегид и сахароза

17. С гидроксидом меди (II) реагируют все вещества ряда

1) CH3COOH, C6H12O6, C12H22O11

2) CH3COOCH3, HCOOH, CH3COOH, 3) CH3CHO, CH3COOH, C2H5COOCH3

4) HCOOH, HCHO, CH3CO–CH2

18. Реакции присоединения характерны для каждого вещества ряда:

1) пропан, бензол, метаналь 2) циклопропан, бутадиен-1,3, бензол 3) бутен-1, гексан, уксусная кислота 4) пропин, этен, бутан19. Гидроксильную группу не содержат молекулы

1) глицерина 2) сложных эфиров 3) глюкозы 4) фенола

20. Олеиновая кислота не реагирует с 1) бромной водой 2) водородом 3) сульфатом натрия 4) этанолом

21. Раствор этанола можно отличить от глицерина с помощью

1) фенолфталеина 2) лакмуса 3) гидроксида меди (II) 4) хлорида натрия

22. Для этилацетата и уксусной кислоты характерна реакция с

1) раствором Na2CO3

2) раствором NaOH 3) Na2O 4) бромной водой

23. При окислении пропанола-2 оксидом меди (II) образуется

1) ацетальдегид 2) пропионовая кислота

88

Page 87: эл курс подг к егэ Word (2)

3) пропаналь 4) пропанон

24. Карбоновые кислоты не взаимодействуют с 1) хлором 2) гидроксидом калия 3) этанолом 4) метаном

25. В схеме превращений

СН3COONa→ Х →СН3CI веществом « Х» является

1) СН3CHO 2) CH3 OH 3) CH4 4) C2H6

26. Спирт, при окислении которого можно получить альдегид, имеет формулу:

1) CH3CHOHCH3

2) CH3CH2CH2OH3) (CH3)3COH4) CH3CHOHCH2CH3

27. В схеме превращенийСН3СООН → Х СН4

веществом «Х» является1) CH3CHO2) CH3CH2OH3) C2H6

4) CH3COONa

28. Глицерин может взаимодействовать с 1) гидроксидом меди(II) 2) сульфатом натрия 3) бромной водой 4) натрием 5) азотной кислотой 6) уксусной кислотой

29. Фенол взаимодействует с 1) гидроксидом натрия 2) серной кислотой 3) бромной водой 4) соляной кислотой 5) метаналем 6) кислородом

89

Page 88: эл курс подг к егэ Word (2)

30. Реакция «серебряного зеркала» характерна для 1) уксусной кислоты 2) метановой кислоты 3) формальдегида 4) фенола 5) этилацетата 6) глюкозы

Занятие 17 Азотсодержащие органические вещества

План изучения темы1). Амины, получение, физические и химические свойства.2). Аминокислоты, классификация, строение, физические и химические свойства. Строение их молекул, способы получения, биологическая роль. Отдельные представители аминокислот: глицин, аланин, серин.

Тесты для самостоятельной работы

1. Число σ-связей в молекуле этиламина равно

1) 6 2) 8 3) 9 4) 10

2. Число σ-связей в молекуле изопропиламина равно

1) 8 2) 9 3) 10 4) 12

3. Оснóвные свойства возрастают в ряду

90

Page 89: эл курс подг к егэ Word (2)

1) метиламин- аммиак- анилин2) аммиак- диметиламин- анилин3) анилин- аммиак- метиламин4) диметиламин- метиламин- аммиак

4. Основные свойства убывают в ряду

1) анилин- диэтиламин- этиламин2) анилин- этиламин- диэтиламин3) диэтиламин- этиламин – анилин4) этиламин- анилин- диэтиламин

5. Этиламин не реагирует с

1) водородом2) водой3) азотистой кислотой4) хлороводородом

6. Водный раствор диметиламина имеет реакцию среды

1) кислую2) слабокислую3) щелочную4) нейтральную

91

Page 90: эл курс подг к егэ Word (2)

7. Анилин в растворе можно обнаружить с помощью

1) натрия2) гидрокарбоната калия3) гидроксида натрия4) бромной воды

8. Число первичных аминов состава C4H11N равно

1) 2 2) 3 3) 4 4) 5

9. Метиламин не реагирует с

1) гидроксидом натрия2) кислородом3) хлороводородом4) водой

10. Для обнаружения в составе белков остатков ароматических аминокислот используют

1) биуретовую реакцию2) ксантопротеиновую реакцию3) реакцию гидролиза4) реакцию поликонденсации

11. При взаимодействии аминокислот между собой образуются

1) соли аминокислот2) новые аминокислоты3) пептиды4) сложные эфиры

12. Аминокислоту можно получить при взаимодействии аммиака с

1) хлорбензолом2) фенолом3) нитробензолом4) 3-хлорпропионовой кислотой

13. Анилин может реагировать с

1) Br2 2) NaOH 3) K2CO3 4) SO3

92

Page 91: эл курс подг к егэ Word (2)

14. Гидролиз белков протекает через стадию образования

1) декстринов2) пептидов3) глицеридов4) аминов

15. Белки приобретают ярко-фиолетовую окраску под действием 1) HNO3 (конц.)2) Cu(OH)2

3) H2SO4 (конц.)4) [Ag(NH3)2]OH

16. Глицин взаимодействует с веществами: 1) хлороводород2) хлорметан3) этан4) этанол5) бутен6) гидроксид калия

17. С раствором гидроксида калия взаимодействуют :

1) метанол 2) метиламин 3) глицин 4) хлорид метиламмония 5) анилин 6) сульфат фениламмония

18. Анилин взаимодействует с

1) гидроксидом натрия 2) серной кислотой 3) бромной водой 4) соляной кислотой 5) этанолом 6) кислородом

19. Аминопропановая кислота взаимодействует с

1) метаналем 2) гидроксидом натрия 3) бромоводородной кислотой 4) хлоридом калия 5) этанолом 6) аланином

93

Page 92: эл курс подг к егэ Word (2)

20. Метиламин взаимодействует с 1) водой 2) этанолом 3) муравьиной кислотой 4) кислородом 5) гидроксидом натрия 6) соляной кислотой

Занятие 18-19.

Задания части 3

1. Используя метод электронного баланса, составьте уравнение реакции:KNO2 + KMnO4 + H2SO4 ® … + … +… .

Определите окислитель и восстановитель.

2. Используя метод электронного баланса, составьте уравнение реакции: K2Cr2O7 + … + H2SO4 ® Fe2 (SO4)3 + …+ Cr2(SO4)3 + H2O Определите окислитель и восстановитель.

3. Используя метод электронного баланса, составьте уравнение реакции:P + KClO3 … + … .

Определите окислитель и восстановитель.

4. Используя метод электронного баланса, составьте уравнение реакции:Fe(OH)2 + KMnO4 + H2O ® … + … .

Определите окислитель и восстановитель.

5. Даны водные растворы хлорида железа (III), хлорида аммония, иодида калия, сульфата меди (II) и хлорида бария. Приведите уравнения четырех возможных реакций между этими веществами.

6. Даны водные растворы: сульфида натрия, нитрата алюминия, перманганата калия, и концентрированная соляная кислота. Приведите уравнения четырех возможных реакций между этими веществами.

7. Даны водные растворы сульфата хрома (III), перманганата калия, карбоната калия, сульфата меди (II) и азотной кислоты. Приведите уравнения четырех возможных реакций между этими веществами.

8. Напишите уравнения реакций, с помощью которых можно осуществить следующие превращения: Нитрит аммония   X1   X2   X3   X4

9. Напишите уравнения реакций, с помощью которых можно

94

Page 93: эл курс подг к егэ Word (2)

осуществить следующие превращения: Этилен   X1   X2   X3    X4   X5

10. Напишите уравнения реакций, с помощью которых можно осуществить следующие превращения:

C6H14   X1   X2   X3   X4 

 X3

11. Напишите уравнения реакций, с помощью которых можно осуществить следующие превращения: Fe   X1   X2   X3   X4

12.Напишите уравнения реакций, с помощью которых можно осуществить следующие превращения: Бензол   X1   X2   X3   X4

   X5

13.Газ, полученный при взаимодействии 8 г меди с 60 г 80%-ого раствора азотной кислоты пропустили на холоде через 25 мл 40%-ого раствора гидроксида = 1,4 г/мл). Рассчитайте массовые доли веществ в образовавшемся растворе.

14. К 20%-ному раствору гидрофосфата натрия массой 106,5 г добавили 8,4 г гидрида натрия. Рассчитайте массовые доли веществ в образовавшемся растворе.

15. Твердый остаток, полученный при термическом разложении 5,64 г нитрата меди (II), поместили в 40,8 г 20%-ного раствора гидросульфата калия. Рассчитайте массовые доли веществ в образовавшемся растворе.

16. Газ, полученный при гидролизе 14,8 г нитрида кальция, пропустили через 66,7 мл 25%-ного раствора серной кислоты (r = 1,176 г/мл). Определите массовую долю вещества в образовавшемся растворе.

17. При взаимодействии 5,8 г предельного альдегида с избытком аммиачного раствора оксида серебра при нагревании образовался осадок массой 2,6 г. Выведите молекулярную формулу альдегида. 18. Неизвестный алкен массой 2,52 г полностью обесцвечивает 300 г 3,2%-ного раствора брома в тетрахлорпентане. Выведите молекулярную формулу алкена.

20. При взаимодействии 13,2 г предельного альдегида с избытком

95

Page 94: эл курс подг к егэ Word (2)

гидроксида меди (II) при нагревании образовался осадок массой 43,2 г. Выведите молекулярную формулу альдегида.

Выходной контроль

Часть 1А1. Число неспаренных электронов в возбужденном состоянии атома фосфора равно

1) 3 2) 5 3) 2 4) 6

А2. Окислительные свойства элементов усиливаются в ряду: 1) S ® P ® Si 2) N ® P ® As 3) C ® N ® P 4) As ® P ® N

А3. Энергия связи уменьшается в ряду 1) HF ® HCI ® HBr 2) HI® HBr ® HCI 3) AsH3 ® PH3 ® NH3

4) O2 ® H2O ® OF2

А4. Высшую валентность марганец проявляет в соединении:

1) Na2MnO4 2) NaMnO4 3)Mn2O3 4) MnO2

А5. К веществам с молекулярной кристаллической решёткой относятся

1) глюкоза, алмаз, железо 2) фторид натрия, кремний, оксид углерода (IV) 3) сахароза, белый фосфор, йод 4) хлорид натрия, бор, гидроксид натрия

А6. Основным и амфотерным оксидом соответственно являются 1) CuO и NO2

2) BeO и Cr2O3

3) Na2O и AI2O3

4) BaO и FeO

А7. Верны ли следующие суждения о свойствах гидроксида железа(III)? А. Гидроксид железа (III) – слабый электролит с амфотерными свойствами. Б. Гидроксид железа (III) – сильный электролит с основными свойствами.

96

Page 95: эл курс подг к егэ Word (2)

1) верно только А 2) верно только Б 3) верны оба суждения 4) оба суждения неверны

А8. Йод является окислителем в реакции

1) хлором 2) азотной кислотой 3) пероксидом водорода 4) сероводородом

А9. Химическая реакция возможна между:

1) FeCl2 и Cu 2) S и P2O5

3) FeCl2 и Cl2

4) BaO и NaOH

А10. Оксид цинка взаимодействует с каждым из двух веществ 1) водой и гидроксидом натрия 2) серной кислотой и гидроксидом калия 3) оксидом магния и сульфатом натрия 4) водой и серной кислотой

А11. Раствор серной кислоты реагирует с каждым веществом ряда:

1) AgNO3, Cu, CuO 2) K2CO3, Be (OH)2, HgO 3) Fe, KOH, H3PO4

4) Na2S, KNO3, Ba(OH)2

А12. Укажите реакцию, в результате которой образуется оксид азота (I) 1) NaNO3

2) Cu(NO3)2

3) NH4NO2

4) NH4NO3

А13. В схеме превращенийBa X1 X2

веществами «X1» и «X2» являются соответственно:

1) BaO и BaCO3

2) Ba (OH)2 и BaCO3

3) Ba (OH)2 и Ba (HCO3)2

97

Page 96: эл курс подг к егэ Word (2)

4) Ba(OH)2 и BaC2

А14. Соединение 2-метилпентен-2 и гексен-1 являются:

1) гомологами 2) структурными изомерами 3) межклассовыми изомерами 4) геометрическими изомерами

А15. Число σ- связей в молекуле бутина-1 равно: 1) 10 2) 9 3) 10 4) 11

А16. Глицерин взаимодействует с каждым веществом пары

1) гидроксидом меди(II) и хлоридом калия 2) натрием и уксусной кислотой 3) натрием и карбонатом натрия 4) гидроксидом меди(II) и метаном

А17. При щелочном гидролизе этилацетата образуются

1) CH3COOH и CH3ONa 2) CH3COOH и C2H5ONa 3) CH3COONa и C2H5OH 4) CH3COONa и C2H5ONa

А18. В схеме превращений СН3COONa→ Х →СН3CI веществом « Х» является

1) СН3CHO 2) CH3 OH 3) CH4 4) C2H6

А19. К реакциям замещения и обмена относятся соответственно 1) С6Н6 + CI2 и CuO + HCI→ 2) C6Н6 + CI2 и NaOH + H2SO4→ 3) CH3COOH + CH3OH → и AgNO3 + NaCI → 4) CH3COOH + NaOH → и СН4 + CI2

А20. Для уменьшения скорости химической реакции2CO + O2 = 2CO2 + Q

необходимо:

1) увеличить концентрацию оксида углерода (II) 2) понизить температуру 3) повысить давление 4) повысить температуру

98

Page 97: эл курс подг к егэ Word (2)

А21. Химическое равновесие в системе:2SO2(г) + O2(г) ⇄ 2SO3(г) + Q

смещается в сторону образования продуктов реакции при

1) повышении давления и понижении температуры 2) понижении давления и понижении температуры 3) повышении давления и повышении температуры 4) понижении давления и повышении температуры

А22. Слабым электролитом является

1) H2SO4

2) Na2SO3

3) NH4OH 4) NaOH

А23. Одновременно в водном растворе могут находиться все ионы ряда

1) Mg2+, Fe3+, Cl–, SO42–

2) Mg2+, Na+, OH–, NO3–

3) Zn2+, Ag+, PO43–, SO3

2–

4) Ba2+, K+, SO42–, F–

А24. Схема превращения S–1 ® S+4 соответствует уравнению реакции:

1) 2PbS + 3O2 = 2PbO + 2SO2

2) H2S + H2SO4(конц) = S¯ + SO2 + 2H2O 3) 4FeS2 + 11O2 = 2Fe2O3 + 8SO2

4) 2H2S + SO2 = 2H2O + 3S¯

А25. Нейтральную среду имеет водный раствор 1) FeCI3

2) ZnCI2

3) Na2CO3

4) NaNO3

А26. Продуктом взаимодействия 2-метилбутана с хлором при освещении является:

1) 1-хлор-2-метилбутан 2) 2- хлор -2-метилбутан 3) 3-хлор-2-метилбутан 4) 1-хлор-2-метилбутен-1

А27. И этанол, и фенол способны реагировать с

1) Na 2) NaOH 3) HBr 4) FeCl3

99

Page 98: эл курс подг к егэ Word (2)

А28. Методом вытеснения воды можно собрать

1) аммиак 2) оксид серы (IV) 3) хлороводород 4) кислород

А29. Производство чугуна – процесс

1) безотходный 2) периодический 3) непрерывный 4) циркуляционный

А30. В соответствии с термохимическим уравнением реакции2NO(г) + O2(г) = 2NO2(г) + 113,7 кДж

22,74 кДж теплоты выделится при окислении оксида азота (II) объёмом

1) 13,44 л 2) 4,48 л 3) 8,96 л 4) 6,72 л

В1. Установите соответствие между названием вещества и его принадлежностью к определенному классу (группе) органических соединений. НАЗВАНИЕ ВЕЩЕСТВА КЛАСС (ГРУППА)

ОРГАНИЧЕСКИХ СОЕДИНЕНИЙ

А) этиленгликоль 1) углевод Б) рибоза 2) многоатомный спирт В) толуол 3) кетон Г) ацетон 4) алкан

5) ароматический углеводород 6)

В2. Установите соответствие между схемой химической реакции и

изменением степени окисления восстановителя.

СХЕМА РЕАКЦИИ ИЗМЕНЕНИЕ СТЕПЕНИ ОКИСЛЕНИЯ ВОССТАНОВИТЕЛЯ А) KNO2 + CI2 + H2O ® KNO3+HCI 1) I- ® I0 Б) HI + H2SO4 ® H2S +I2 + H2O 2) CI+5® CI-

В) H2S + CI2+ H2O ® H2SO4 + HCI 3) CI0® CI- Г) KCIO3 + KI + HCI ® I2 + KCI + H2O 4) S-2 ® S+6

100

Page 99: эл курс подг к егэ Word (2)

5) S+6 ® S -2

6) N+3 ® N+5

В3.Установите соответствие между формулой вещества и продуктами электролиза его водного раствора.

ФОРМУЛА ВЕЩЕСТВА ПРОДУКТЫ ЭЛЕКТРОЛИЗА А) BaCl2 1) Fe, H2, O2

Б) Fe(NO3)2 2) H2, O2

В) Na2SO4 3) Cu, Cl2, O2

Г) CuCl2 4) H2, Cl2

5) Ba, Cl2

6) Cu, Cl2

В4. Установите соответствие между названием соли и окраской лакмуса в её водном растворе.

НАЗВАНИЕ СОЛИ ОКРАСКА ЛАКМУСА В ВОДНОМ РАСТВОРЕ СОЛИ А) нитрат цинка 1) желтая Б) нитрат бериллия 2) фиолетовая В) карбонат лития 3) синяя Г) иодид натрия 4) красная

В5. И цинк и гидроксид натрия способны реагировать с

1) Cl2

2) NaCl 3) HCl 4) BaCl2

5) FeCl2

6) Ba (OH)2

В6. Для этина характерны:

1) sp-гибридизация орбиталей атомов углерода в молекуле 2) sp3-гибридизация орбиталей атомов углерода в молекуле 3) окисление раствором перманганата калия 4) хорошая растворимость в воде 5) реакция гидратации 6) реакция с аммиачным раствором оксидом серебра (I)

В7. Фенол взаимодействует с

1) гидроксидом натрия

101

Page 100: эл курс подг к егэ Word (2)

2) серной кислотой 3) бромной водой 4) соляной кислотой 5) метаналем 6) кислородом

В8. Метиламин может реагировать с

1) гидроксидом натрия 2) серной кислотой 3) бромной водой 4) азотистой кислотой 5) этанолом 6) кислородом

В9. При взаимодействии 2 моль цинка и 200 г 19,6%-ного раствора серной кислоты образуется соль массой _________ г. (Запишите число с точностью до целых.)

В10. При горении 40л аммиака в кислороде объёмом 60л остаётся избыток кислорода объёмом (н.у.) _________ л. (Запишите число с точностью до целых.)

Часть 3

С1. Используя метод электронного баланса, составьте уравнение реакции:HIO3 + H2O2 ® I2… + … .

Определите окислитель и восстановитель.

С2. Даны вещества: медь, хлорид железа(III), гидроксид натрия и азотная кислота (конц.). Приведите уравнения четырех возможных реакций между этими веществами.С3. Напишите уравнения реакций, с помощью которых можно осуществить следующие превращения:

(CH3CO)2O   X1   X2   X3   X4   X3

С4. К 20%-ному раствору гидрофосфата натрия массой 106,5 г добавили 8,4 г гидрида натрия. Рассчитайте массовые доли веществ в образовавшемся растворе.

С5. При взаимодействии 5,8 г предельного альдегида с избытком аммиачного раствора оксида серебра при нагревании образовался осадок массой 21,6 г. Выведите молекулярную формулу альдегида.

102

Page 101: эл курс подг к егэ Word (2)

ОтветыВходной контроль

Часть 1А1 А2 А3 А4 А5 А6 А7 А8 А9 А10 1 4 3 2 4 2 3 2 1 3

А11 А12 А13 А14 А15 А16 А17 А18 А19 А202 4 3 3 3 4 3 2 4 3

А21 А22 А23 А24 А25 А26 А27 А28 А29 А304 1 3 3 3 2 2 3 4 3

Часть 2№ Ответ № Ответ

B1 3416 B6 245B2 3562 B7 236B3 3455 B8 346B4 3212 B9 15

103

Page 102: эл курс подг к егэ Word (2)

B5 126 B10 20

Часть 3С1. Используя метод электронного баланса, составьте уравнение реакции: K2Cr2O7 + HCI ® CrCI3 + …+ … + H2O Определите окислитель и восстановитель.Ответ: K2Cr2O7 + 14HCI = 2CrCI3 + 3CI2 +2KCI + 7H2O 3 2CI– –2ē → CI2

0

1 2Cr+6 +6ē → 2Cr+3

С2. Даны вещества: сульфат меди (II), иодид калия, гидрокарбонат натрия, гидроксид натрия. Напишите уравнения четырех возможных реакций между этими веществами.

Ответ : 1) 2CuSO4+ 4KI = 2CuI + I2 +2K2SO4

2) CuSO4+2NaHCO3= Cu(OH)2¯ +Na2SO4 +2CO2

3) CuSO4+2NaOH = Cu(OH)2 ¯+ Na2SO4

4) NaHCO3+NaOH = Na2CO3 + H2OС3. Напишите уравнения реакций, с помощью которых можно осуществить следующие превращения: C2H2   X1   X2   X3   X4

 X5

Ответ : 1) 3C2H2 C6H6

2) C6H6 + C2H5Cl C6H5CH2CH3 + HCl3) C6H5 – CH2CH3 + Cl2 ® C6H5 – CHClCH3 + HCl4) C6H5 – CHClCH3 + КOH(спирт.) ® C6H5CH = CH2+ KCl + H2O5) 3C6H5CH = CH2 + 2KMnO4 + 4H2O ® ® 3C6H5CH(OH)CH2OH + 2MnO2 + 2KOHС4. Оксид фосфора (V), полученный при окислении 3,1 г фосфора, поместили в 25 мл 25%-ного раствора гидроксида натрия (r=1,28г/мл). Определите состав образовавшейся соли и её массовую долю (в %) в растворе.Ответ:1) Составим уравнение реакции и рассчитаем количество вещества образовавшегося оксида фосфора (V):4P + 5O2 = 2P2O5

n(P) =3,1/31 =0,1 (моль); n(P2O5)= 1/2 n(P) = 0,05(моль)2) Определим соотношение количества вещества оксида фосфора (V) и гидроксида натрия: n(NaOH) = 25 ´ 1,28´0,25/40 = 0,2 (моль)n(P2O5) : n(NaOH) = 0,05 : 0,2=1:4; NaOH – избыток3) Составим уравнение реакции и определим массу образовавшейся соли:P2O5 + 4 NaOH =2Na2HPO4 + H2On(Na2HPO4)=2n(P2O5)=0,1 (моль); m(Na2HPO4)= 0,1´142 =14,2 (г)4) Рассчитаем массу раствора и массовую долю соли в нем:

104

Page 103: эл курс подг к егэ Word (2)

m(р-ра) = m(25%-ного р-ра NaОH) + m(P2O5) = 32 + 0,05´142 = 39,1 (г).w(Na2HPO4) = 14,2/39,1 = 0,363 или 36,3%.

С5. При взаимодействии 18 г неизвестной одноосновной карбоновой кислоты с избытком гидрокарбоната натрия выделилось 6,72л газа (н.у.). Установите молекулярную формулу кислоты.Ответ:1) Запишем уравнение реакции в общем виде и найдем количество вещества кислоты:СnH2n+1COOH + NaHCO3 → СnH2n+1COONa + CO2 + H2O n(CnH2n+1COOH) = n(CO2) = 6,72/22,4 = 0,3 (моль). 2) Рассчитаем молярную массу кислоты:М(CnH2n+1CООH) = 18/0,3 = 60 (г/моль)3) Определим число атомов углерода в молекуле кислоты и установим её формула: М(CnH2n+1CООH) = 12n + 2n +1 + 12 + 16´2 + 1 = 14n + 4614n + 46 = 60; n = 1. Формула кислоты CH3CООH.

Тема. Строение атома

А1 А2 А3 А4 А5 А6 А7 А8 А9 А101 4 3 1 4 3 1 3 1 4

А11 А12 А13 А14 А15 А16 А17 А18 А19 А202 2 4 2 4 4 3 2 4 3

А21 А22 А23 А24 А25 А26 А27 А28 А29 А301 4 1 4 4 2 2 2 3 3

Тема. Периодический закон и периодическая система химических элементов Д.И. Менделеева

А1 А2 А3 А4 А5 А6 А7 А8 А9 А103 2 1 4 2 1 2 4 1 2

А11 А12 А13 А14 А15 А16 А17 А18 А19 А202 4 3 4 2 4 1 2 4 4

А21 А22 А23 А24 А25 А26 А27 А28 А29 А301 2 2 1 3 1 3 3 3 4

Тема. Химическая связь и кристаллические решетки

А1 А2 А3 А4 А5 А6 А7 А8 А9 А101 4 3 2 3 2 1 3 3 1

А11 А12 А13 А14 А15 А16 А17 А18 А19 А202 4 3 4 2 4 1 2 4 4

А21 А22 А23 А24 А25 А26 А27 А28 А29 А303 3 1 1 4 1 3 1 3 2

105

Page 104: эл курс подг к егэ Word (2)

Тема. Валентность и степень окисления

А1 А2 А3 А4 А5 А6 А7 А8 А9 А104 4 4 3 1 3 1 4 1 1

А11 А12 А13 А14 А15 А16 А17 А18 А19 А201 1 1 4 3 3 1 3 3 1

А21 А22 А23 А24 А25 А26 А27 А28 А29 А304 4 1 2 1 3 3 2 4 2

Тема. Химические реакции

А1 А2 А3 А4 А5 А6 А7 А8 А9 А103 3 1 2 2 4 3 3 3 3

А11 А12 А13 А14 А15 А16 А17 А18 А19 А202 4 1 3 1 3 2 3 2 3

А21 А22 А23 А24 А25 А26 А27 А28 А29 А301 3 4 1 1 4 2 2 4 4

Тема. Тепловой эффект химической реакции

А1 А2 А3 А4 А5 А6 А7 А8 А9 А101 2 3 3 2 4 2 3 4 4

А11 А12 А13 А14 А15 А16 А17 А18 А19 А203 2 2 3 4 3 3 2 1 4

А21 А22 А23 А24 А25 А26 А27 А281 1 2 2 1 3 3 2

Тема. Скорость химической реакции. Химическое равновесие

А1 А2 А3 А4 А5 А6 А7 А8 А9 А102 1 1 3 3 3 3 4 3 2

А11 А12 А13 А14 А15 А16 А17 А18 А19 А203 4 2 4 1 2 1 4 4 2

А21 А22 А23 А24 А25 А26 А27 А28 А29 А304 2 1 1 2 1 1 4 3 4

Тема. Растворы. Электролитическая диссоциация

А1 А2 А3 А4 А5 А6 А7 А8 А9 А101 4 3 1 3 2 3 2 2 3

А11 А12 А13 А14 А15 А16 А17 А18 А19 А201 4 4 4 4 1 3 1 3 4

А21 А22 А23 А24 А25 А26 А27 А28 А29 А3010 64 11.2 1,8 70 30 10 18 10 5

106

Page 105: эл курс подг к егэ Word (2)

Тема. Важнейшие классы неорганических соединений

А1 А2 А3 А4 А5 А6 А7 А8 А9 А104 3 3 4 4 2 4 2 1 4

А11 А12 А13 А14 А15 А16 А17 А18 А19 А204 3 2 2 3 2 1 3 4 4

А21 А22 А23 А24 А25 А26 А27 А28 А29 А306531 6231 3146 3142 1254 1452 2345 136 145 36

Тема. Гидролиз неорганических и органических соединений

А1 А2 А3 А4 А5 А6 А7 А8 А9 А102 4 3 3 3 3 1 3 1 2

А11 А12 А13 А14 А15 А16 А17 А18 А19 А203 3 4 4 4 3 4 2 3 1

А21 А22 А23 А24 А25 А262 60 98 28 64 64

27. 2Fe(NO3)3 + 3Na2CO3 = Fe2(CO3)3 + 6NaNO3

При взаимодействии растворов нитрата железа (III) и карбоната натрия образуется соль – карбонат железа (III), которая подвергается полному (необратимому) гидролизу с образованием осадка и выделением газа: 2Fe3+ + 3CO3

2- + 3H2O = 2Fe(OH)3↓ + 3CO2↑ В итоге молекулярное уравнение записывается так: 2Fe(NO3)3 + 3Na2CO3 + 6H2O = 2Fe(OH)3↓ + 6NaNO3 + 3CO2↑ +3H2O или 2Fe(NO3)3 + 3Na2CO3 + 3H2O = 2Fe(OH)3↓ + 6NaNO3 + 3CO2↑28. 2CrCl3 + 3Na2S = Cr2S3 + 6NaCl

При взаимодействии растворов двух данных солей образуется соль- сульфид хрома (III), которая подвергается полному гидролизу с образованием осадка и выделением газа:

2Cr3+ + 2S2- + 6H2O = 2Cr(OH)3↓ + 3 H2S↑Итоговое молекулярное уравнение реакции записывается так:

2CrCl3 + 3Na2S + 6H2O = 2Cr(OH)3↓+ 6NaCl + 3H2S↑

29. 2FeCl3 + 3K2CO3 = Fe2(CO3)3 + 6KNO3

При взаимодействии растворов хлорида железа (III) и карбоната калия образуется соль – карбонат железа (III), которая подвергается полному (необратимому) гидролизу с образованием осадка и выделением газа : 2Fe3+ + 3CO3

2- + 3H2O = 2Fe(OH)3↓ + 3CO2↑ В итоге молекулярное уравнение записывается так: 2FeCl3 + 3K2CO3 + 6H2O = 2Fe(OH)3↓ + 6KNO3 + 3CO2↑ +3H2O или 2FeCl3 + 3K2CO3 + 3H2O = 2Fe(OH)3↓ + 6KNO3 + 3CO2↑

107

Page 106: эл курс подг к егэ Word (2)

30. 2Al(NO3)3 + 3Na2CO3 = Al2(CO3)3 + 6NaNO3

Взаимодействие данных солей приводит к образованию карбоната алюминия, который подвергается полному гидролизу с образованием осадка и выделением газа: 2Al3+ + 3CO3

2- + 3H2O = 2Al(OH)3↓ + 3CO2↑ Итоговое молекулярное уравнение реакции записывается так: 2Al(NO3)3 + 3Na2CO3 + 3H2O = Al(OH )3↓ + 6NaNO3 + 3CO2↑

Тема. Металлы

А1 А2 А3 А4 А5 А6 А7 А8 А9 А102 1 4 3 4 2 2 13 3 1

А11 А12 А13 А14 А15 А16 А17 А18 А19 А202 2 1 2 4 2 4 3 3 4

А21 А22 А23 А24 А25 А26 А27 А28 А29 А301223 1122 2443 3113 3121 2362 4122 3211 4,48 4,48

Тема. НеметаллыА1 А2 А3 А4 А5 А6 А7 А8 А9 А104 3 3 1 1 4 3 2 2 3

А11 А12 А13 А14 А15 А16 А17 А18 А19 А203 4 2 4 3 3 3 2 3 14

А21 А22 А23 А24 А254 2 2 1 2

Тема. Углеводороды

А1 А2 А3 А4 А5 А6 А7 А8 А9 А104 4 1 2 1 4 1 2 2 1

А11 А12 А13 А14 А15 А16 А17 А18 А19 А202 2 1 2 1 3 4 2 3 3

А21 А22 А23 А24 А25 А26 А27 А28 А29 А303 3 245 1356 235 124 245 245 246 1245

Тема. Кислородсодержащие органические соединения

А1 А2 А3 А4 А5 А6 А7 А8 А9 А102 2 3 4 3 2 4 3 2 3

А11 А12 А13 А14 А15 А16 А17 А18 А19 А202 2 3 3 2 3 1 2 2 3

А21 А22 А23 А24 А25 А26 А27 А28 А29 А303 2 4 4 3 2 4 1456 1356 236

108

Page 107: эл курс подг к егэ Word (2)

Тема. Азотсодержащие органические соединения

А1 А2 А3 А4 А5 А6 А7 А8 А9 А103 4 3 3 1 3 4 3 1 2

А11 А12 А13 А14 А15 А16 А17 А18 А19 А203 4 1 2 2 146 346 2346 2356 146

Тема. Задания части С

1. 5KNO2 + 2KMnO4 + 3H2SO4 ® 5KNO3 + K2SO4 + 2MnSO4 + 3H2O

5 N+3– 2ē → N+5

2 Mn+7 + 5ē → Mn+2

Марганец в степени окисления +7 (или перманганат калия за счёт марганца в степени +7) является окислителем, а азот в степени окисления +3 (или нитрит калия за счет азота в степени окисления +3) – восстановителем.

2. K2Cr2O7 + 6FeSO4 + 7H2SO4 ® 3Fe2 (SO4)3 + K2SO4+ Cr2(SO4)3 + 7H2O 6 Fe+2 –1ē → Fe+3

1 2Cr+6 +6ē → 2Cr+3

Железо в степени окисления +2 является восстановителем, а хром в степени окисления +6 (или дихромат калия за счет хрома в степени окисления +6) – окислителем.

3. 6P + 5KClO3 5KCl + 3P2O5

6 Р0 − 5ē → Р+5

5 Cl+5 + 6ē → Cl–1

Фосфор в степени окисления 0 является восстановителем, а хлор в степени окисления +5 (или хлорат калия за счет хлора в степени окисления +5) – окислителем.

4. 3Fe(OH)2 + KMnO4 + 2H2O = 3Fe(OH)3 + MnO2+KOH

3 Fe+2 − 1ē = Fe+3

1 Mn+7 + 3ē = Mn+4

Железо в степени окисления +2 является восстановителем, а марганец в степени окисления +7 (или перманганат калия за счет марганца в степени окисления +7) – окислителем.

109

Page 108: эл курс подг к егэ Word (2)

5. 1) 6FeCl3 + 2NH4Cl = 6FeCl2 + 8HCl + N2

2) 2FeCl3 + 2KI = 2FeCl2 + I2 + 2KCl 3) 2CuSO4 + 4KI = 2CuI¯ + I2 + 2K2SO4

4) CuSO4 + BaCl2 = BaSO4¯ + CuCl2

6. 1) 3Na2S + 2Al(NO3)3 + 6H2O = 6NaNO3 + 2Al(OH)3↓ + 3H2S↑ 2) 3Na2S + 2KMnO4 + 4H2O = 3S↓ + 2MnO2 + 6NaOH + 2KOH 3) 2KMnO4 + 16HCl = 2KCl + 2MnCl2 + 5Cl2 + 8H2O 4) Na2S + 2HCl = 2NaCl + H2S↑

7. 1) Cr2(SO4)3 + 2KMnO4 + 7H2O = K2Cr2O7 + 2Mn(OH)4¯ + 3H2SO4

2) Cr2(SO4)3 + 3K2CO3 + 3H2O = 2Cr(OH)3¯ + 3K2SO4 + 3CO2

3) K2CO3 + CuSO4 = CuCO3¯ + K2SO4

4) K2CO3 + 2HNO3 = 2KNO3 + H2O + CO2

8. Нитрит аммония   X1   X2   X3   X4

1) NH4NO2 N2 + 2H2O 2) N2 + 3Mg Mg3N2

3) Mg3N2 + 6H2O = 3Mg(OH)2¯ + 2NH3

4) 4NH3 + 5O2 4NO + 6H2O

9. Напишите уравнения реакций, с помощью которых можно осуществить следующие превращения:Этилен   X1   X2   X3    X4   X5

Ответ: 1) Н2С = СН2 + Сl2 H2C = CH + HCl׀ Cl2) H2C = CH + KOH(спирт. р-р) ® HС ≡ CH + KCl + H2O׀ Cl3) HС ≡ CH + H2O CH3 − CHO4) 5CH3 − CHO + 2KМnO4 + 3H2SO4 → 5CH3COOH + 2MnSO4+ + K2SO4 + 3H2O5) CH3COOH + C2H5OH CH3COOC2H5 + H2O

10. C6H14   X1   X2   X3   X4 

 X3

1) CH3 – (CH2)4 – CH3 C6H6 + 4H2

2) C6H6 + Cl2 C6H5Cl + HCl 3) C6H5Cl + H2O C6H5OH + HCl

110

Page 109: эл курс подг к егэ Word (2)

4) C6H5OH + NaOH ® C6H5ONa + H2O 5) C6H5ONa + CO2 + H2O ® C6H5OH + NaHCO3

11. Fe   X1   X2   X3   X4

1) 2Fe + 3Cl2 = 2FeCl3

2) 2FeCl3 + H2S = 2FeCl2 + S¯ + 2HCl 3) 5FeCl2 + KMnO4 + 8HCl = 5FeCl3 + MnCl2 + KCl + 4H2O 4) 2FeCl3 + Fe = 2FeCl2

12. Бензол   X1   X2   X3   X4

   X5

1) С6Н6 + С2H5Cl C6H5 – CH2CH3 + HCl 2) C6H5CH2CH3 + Br2 C6H5CHBr – CH3 + HBr 3) C6H5CHBrCH3 + KOH C6H5CHOH – CH3 + KBr 4) C6H5CHOHCH3 C6H5CH = CH2 + H2O 5) C6H5CH = CH2 + 2KMnO4 + 3H2SO4 ® C6H5COOH + CO2 + +K2SO4 + 2MnSO4 + 4H2O13. Газ, полученный при взаимодействии 8 г меди с 60 г 80%-ого раствора азотной кислоты пропустили на холоде через 25 мл 40%-ого раствора гидроксида калия (r = 1,4 г/мл). Рассчитайте массовые доли веществ в образовавшемся растворе.Решение. Составим уравнение реакции:Cu + 4HNO3(конц) = Cu(NO3)2 + 2NO2 + 2H2OРассчитаем количество вещества образовавшегося оксида азота (IV):

n(Cu) = 8/64 = 0,125 моль; n(НNO3) = = 0,76 моль,

НNO3 – избыток; n(NO2) = 2n(Cu) = 0,25 моль  Составим уравнение реакции и определим количество вещества KOH:2NO2 + 2KOH = KNO3 + KNO2 + H2On(KOH) = 25 ´ 1,4 ´ 0,4/56 = 0,25 моль3) Определим массы KNO3 и KNO2

n(KNO3) = n(KNO2) = ½n(NO2) = 0,125 мольm(KNO3) = 101 ´ 0,125 = 12,625 гm(KNO2) = 85 ´ 0,125 = 10,625 г4) Определим массу раствора и массовые доли солей в раствореm(р-ра) = m(NO2) + m(р-ра KOH) = 46 ´ 0,25 + 35 = 46,5 гw(KNO3) = 12,625/46,5 = 0,217 или 21,7%w(KNO2) = 10,625/46,5 = 0,228 или 22,8%

14. К 20%-ному раствору гидрофосфата натрия массой 106,5 г добавили 8,4 г гидрида натрия. Рассчитайте массовые доли веществ в образовавшемся растворе.

Решение

111

Page 110: эл курс подг к егэ Word (2)

Составим уравнение реакции и рассчитаем количество вещества образовавшегося гидроксида натрия:NaH + H2O = NaOH + H2

n(NaH) = 8,4/24 = 0,35 (моль)n(NaOH) = n(NaH) = 0,35 (моль)2) Определим соотношение количества вещества гидрофосфата натрия и гидроксида натрия:n(Na2HPO4) = 106,5 ´ 0,2/142 = 0,15 (моль)n(Na2HPO4) : n(NaOH) = 0,15 : 0,35; NaOH – избыток3) Составим уравнение реакции и определим массы веществ в растворе:NaHPO4 + NaOH = Na3PO4 + H2Om(Na3PO4) = 164 ´ 0,15 = 24,6 (г)m(NaOH) = 40 ´ 0,2 = 8 (г)4) Рассчитаем массу раствора и массовые доли веществ в нем:m(р-ра) = m(р-ра Na2HPO4) + m(NaH) – m(H2) =106,5 + 8,4 – 2´0,35=114,2 (г)w(Na2HPO4) = 24,6/114,2 = 0,215 или 21,5%w(NaOH) = 8/114,2 = 0,07 или 7%

15. 1) Запишем уравнение реакции и рассчитаем количество вещества образовавшегося оксида меди (II):2Cu(NO3)2 = 2CuO + 4NO2 + O2

n(Cu(NO3)2) = 5,64/188 = 0,03 (моль)n(CuO) = n(Cu(NO3)2) = 0,03 (моль)2) Запишем уравнение реакции, рассчитаем количества веществ, образовавшихся в результате реакции.2KHSO4 + CuO = K2SO4 + CuSO4 + H2On(KHSO4) = 40,8 ´ 0,2/136 = 0,06 (моль)n(KHSO4) : n(CuO) = 0,06 : 0,03 = 2 : 1n(K2SO4) = n(CuSO4) = n(CuO) = 0,03 (моль)3) Рассчитаем массы образовавшихся солей.m(K2SO4) = 174 ´ 0,03 = 5,22 (г)m(NaOH) = 160 ´ 0,03 = 4,8 (г)4) Рассчитаем массу раствора и массовые доли веществ в нем:m(р-ра) = m(р-ра KHSO4) + m(CuO) = 40,8 + 0,03 ´ 80 = 43,2 (г)w(K2SO4) = 5,22/43,2 = 0,12 или 12%; w( CuSO4) = 4,8/43,2 = 0,11 или 11%

16. Газ, полученный при гидролизе 14,8 г нитрида кальция, пропустили через 66,7 мл 25%-ного раствора серной кислоты (r = 1,176 г/мл). Определите массовую долю вещества в образовавшемся растворе. Решение1) Запишем уравнение реакции и определим количество вещества образовавшегося аммиака.Ca3N2 + 6H2O = 3Ca(OH)2 + 2NH3

n(Ca3N2) = 14,8/148 = 0,1 (моль)

112

Page 111: эл курс подг к егэ Word (2)

n(NH3) = 2n(Ca3N2) = 0,2 (моль)2) Определим соотношение количества вещества аммиака и количества вещества взятой серной кислоты2NO2 + 2KOH = KNO3 + KNO2 + H2On(H2SO4) = 66,7 ´ 1,176 ´ 0,25/98 = 0,2 (моль)n(NH3) : n(H2SO4) = 0,2 : 0,2 = 1 : 13) Составим уравнение реакции и определим массу образовавшейся соли.NH3 + H2SO4 = NH4HSO4 ; m(NH4HSO4) = 115 ´ 0,2 = 23 (г)4) Рассчитаем массу раствора и массовую долю соли в нем.m(р-ра) = m(NH3) + m(р-ра H2SO4) = 0,2 ´ 17 + 78,4 = 81,8 (г)

w(NH4HSO4) = = 0,281 или 28,1%

17. При взаимодействии 5,8 г предельного альдегида с избытком аммиачного раствора оксида серебра при нагревании образовался осадок массой 21,6 г. Выведите молекулярную формулу альдегида. 1) Запишем уравнение реакции в общем виде и найдем количество вещества альдегида:CnH2n+1CHO + [Ag(NH3)2]OH CnH2n+1CHOOH + Ag¯ + + H2O + NH3

n(CnH2n+1CHO) = ½n(Ag) = = 0,1 (моль)

2) Рассчитаем молярную массу альдегида:

М(CnH2n+1CHO) = = 58 (г/моль)

3) Определим число атомов углерода в молекуле альдегида и установим его формулу: М(CnH2n+1CHO) = 12n + 2n +1 + 12 +1 + 16 = 14n + 3014n + 30 = 58n = 2C2H5CHO

18. Неизвестный алкен массой 2,52 г полностью обесцвечивает 300 г 3,2%- ного раствора брома в тетрахлорпентане. Выведите молекулярную формулу алкена. Решение1) Запишем уравнение реакции в общем виде и найдем количество вещества алкена.CnH2n + Br2 ® CnH2nBr2

n(Br2) = 300 ´ 0,032/160 = 0,06 (моль)

2) Рассчитаем молярную массу алкена: М(CnH2n) = = 42 (г/моль)

3) Определим число атомов углерода в молекуле алкена и установим его формулу:М(CnH2n) = 14n 14n = 42

113

Page 112: эл курс подг к егэ Word (2)

20. При взаимодействии 13,2 г предельного альдегида с избытком гидроксида меди (II) при нагревании образовался осадок массой 43,2 г. Выведите молекулярную формулу альдегида.Решение.

Запишем уравнение реакции в общем виде и найдем количество вещества альдегида:CnH2n+1CHO + 2Cu(OH)2 = CnH2n+1COOH + Cu2O + 2H2O

n(CnH2n+1CHO) = n(Cu2O) = = 0,3 моль

2) Рассчитаем молярную массу альдегида:

М(CnH2n+1CHO) = = 44 г/моль

3) Определим число атомов углерода в молекуле альдегида и установим его формулу:М(CnH2n+1CHO) = 12n + 2n + 1 +12 + 1 + 16 = 14n + 3014n + 30 = 44n = 1CH3CHO

Выходной контроль

А1 А2 А3 А4 А5 А6 А7 А8 А9 А10 2 4 1 2 3 3 1 4 3 2

А11 А12 А13 А14 А15 А16 А17 А18 А19 А20 2 4 3 2 2 2 3 3 2 2

А21 А22 А23 А24 А25 А26 А27 А28 А29 А30 1 3 1 3 4 2 1 4 3 3

Часть 2№ Ответ № Ответ

B1 2153 B6 1356B2 6141 B7 1356B3 4126 B8 246B4 4432 B9 64B5 135 B10 30

ЧАСТЬ 3

С1. Ответ: 2HIO3 + 5H2O2 = I2 + 5O2 + 6H2O1 2I+5 + 10ē → I2

0

5 2O–1 – 2ē → O20

Иод в степени окисления +5 (или иодноватая кислота за счёт иода в степени +5) является окислителем, а кислород в степени окисления –1 (или пероксид водорода за счет кислорода в степени окисления –1) – восстановителем.

114

Page 113: эл курс подг к егэ Word (2)

С2. Даны вещества: медь, хлорид железа(III), гидроксид натрия и азотная кислота (конц.).Приведите уравнения четырех возможных реакций между этими веществами.Ответ:

1) Cu + 4HNO3 (конц.) = Cu(NO3)2 + 2NO2 + 2H2O2) Cu + 2FeCI3 = CuCI2 + 2FeCI2

3) HNO3 + NaOH = NaNO3 + H2O 4) FeCI3 + 3 NaOH = Fe(OH)3¯ + 3NaCI

С3.

(CH3CO)2O   X1   X2   X3   X4   X5

Ответ : 1) (CH3CO)2O + H2O ® 2CH3COOH2) CH3COOH + CH3OH CH3COO – CH3 + H2O3) CH3COO – CH3 + NaOH ® CH3COONa + CH3OH4) CH3COONa + NaOH CH4 + Na2CO3

5) 2CH4 C2H2 + 3H2

С4. К 20%-ному раствору гидрофосфата натрия массой 106,5 г добавили 8,4 г гидрида натрия.Рассчитайте массовые доли веществ в образовавшемся растворе.Ответ:1) Составим уравнение реакции и рассчитаем количество вещества образовавшегося гидроксида натрия:NaH + H2O = NaOH + H2

n(NaH) = 8,4/24 = 0,35 (моль); n(NaOH) = n(NaH) = 0,35 (моль)2) Определим соотношение количества вещества гидрофосфата натрия и гидроксида натрия:n(Na2HPO4) = 106,5 ´ 0,2/142 = 0,15 (моль)n(Na2HPO4) : n(NaOH) = 0,15 : 0,35; NaOH – избыток3) Составим уравнение реакции и определим массы веществ в растворе:Na2HPO4 + NaOH = Na3PO4 + H2Om(Na3PO4) = 164 ´ 0,15 = 24,6 (г); m(NaOH изб.) = 40 ´ 0,2 = 8 (г)4) Рассчитаем массу раствора и массовые доли веществ в нем:m(р-ра) = m(р-ра Na2HPO4) + m(NaH) – m(H2) = 106,5 + 8,4 – 2´0,35= 114,2 (г)w(Na2HPO4) = 24,6/114,2 = 0,215 или 21,5%w(NaOH) = 8/114,2 = 0,07 или 7%

С5. При взаимодействии 5,8 г предельного альдегида с избытком аммиачного раствора оксида серебра при нагревании образовался осадок массой 21,6 г.

115

Page 114: эл курс подг к егэ Word (2)

Выведите молекулярную формулу альдегида.Ответ:1) Запишем уравнение реакции в общем виде и найдем количество вещества альдегида:CnH2n+1CHO + 2[Ag(NH3)2]OH CnH2n+1COONH4 + 2Ag¯ + + H2O + 3NH3

n(CnH2n+1CHO) = ½n(Ag) = = 0,1 (моль)

2) Рассчитаем молярную массу альдегида:

М(CnH2n+1CHO) = = 58 (г/моль)

3) Определим число атомов углерода в молекуле альдегида и установим его формулу:М(CnH2n+1CHO) = 12n + 2n +1 + 12 +1 + 16 = 14n + 3014n + 30 = 58n = 2C2H5CHO

116

Page 115: эл курс подг к егэ Word (2)

СодержаниеПрограмма элективного курса по подготовке учащихся кЕдиному экзамену по химии……………………………………………….. 3

Занятие 1. Кодификатор элементов содержания по химии для составления контрольных измерительных материалов единого государственного экзамена 2009 г……………………………….. 8

Занятие 2. Входной контроль……………………………………………….11

Занятие 3. Строение атома………………………………………………… 18

Занятие 4. Периодический закон периодическая система химическихэлементов Д.И. Менделеева………………………………………………. 26

Занятие 5. Химическая связь и кристаллические решетки……………… 30

Занятие 6. Валентность и степень окисления……………………………. 39

Занятие 7. Химические реакции……………………………………………43

Занятие 8. Тепловой эффект химической реакции………………………. 48

Занятие 9. Скорость химических реакций. Химическое равновесие…… 52

Занятие 10. Растворы. Электролитическая диссоциация…………………57

Занятие 11. Важнейшие классы неорганических соединений…………... 61

Занятие 12. Гидролиз неорганических и органических соединений…… 68

Занятие 13. Металлы………………………………………………………. 72

Занятие 14. Неметаллы…………………………………………………….. 77

Занятие 15. Углеводороды………………………………………………….78

Занятие 16. Кислородсодержащие вещества………………………………87

Занятие 17. Азотсодержащие вещества……………………………………91

117

Page 116: эл курс подг к егэ Word (2)

Занятие 18-19. Задания части С…………………………………………….95

Занятие 20. Выходной контроль……………………………………………97Ответы………………………………………………………………………105

Для заметок

118

Page 117: эл курс подг к егэ Word (2)

Для заметок

119

Page 118: эл курс подг к егэ Word (2)

Для заметок

120